<<

May 2010 An Evidence-Based Approach To Volume 12, Number 5 Traumatic Ocular Emergencies Authors Janet Alteveer, MD, FACEP Associate Professor of Emergency Medicine, Robert Wood A 21-year-old male presents to the ED complaining that his vision seems Johnson Medical School, Camden, University of Medicine “wavy,” as if something is jiggling inside his . One hour earlier he was and Dentistry of New Jersey; Faculty and Staff , Emergency Department, Cooper University Hospital, standing on the street when he heard what sounded like a gun firing. He Camden, NJ felt “something funny” in his eye, but there was no blood and no real . Brian Lahmann, MD, MHA On exam there is no periorbital swelling and no obvious trauma to the eye Staff Physician, Department of Emergency Medicine, The itself. His visual acuity is 20/40 in the right eye and 20/30 in the left. Fur- Reading Hospital and Medical Center, Reading, PA ther examination of the right eye reveals a small gray mark in the just Peer Reviewers lateral to the . You wonder if this could be an entry wound and, if so, Adhi Sharma, MD, FACMT, FACEP what would be the best way to detect an intraocular foreign body (IOFB). Chairman, Department of Emergency Medicine, Good Samaritan Hospital Medical Center, West Islip, NY You also wonder whether you should administer an antibiotic. Timothy Walther, MD While you are thinking about the young man’s problem, another Instructor of Emergency Medicine, Elmhurst Hospital Center, patient with an eye complaint arrives in the ED. A 55-year-old woman Mount Sinai School of Medicine, New York, NY presents with pain in her eye after walking along a wooded trail. She thinks CME Objectives that she felt something fly into her left eye, and the pain started after she Upon completion of this article, you should be able to: rubbed her eye. Back at home, she rinsed her eye out with some water and 1. Identify common traumatic eye and disorders. OTC eye drops, with no relief. She reports excruciating, 10/10 pain and 2. Develop a systematic history and physical examination tearing; the bright light bothers her, and she claims she cannot tolerate an for traumatic eye injuries. 3. Initiate emergency evaluation for blunt and penetrating eye exam. You wonder what you can do to facilitate her evaluation and why eye trauma. her injured eye hurts more when the light hits her uninjured eye. 4. Initiate emergency treatment for blunt and penetrating eye trauma.

ye emergencies account for almost 3% of all ED visits in the Date of original release: May 1, 2010 EUnited States1 and 1.4% of all ED visits involving an .2 Date of most recent review: December 1, 2009 Termination date: May 1, 2013 It is exceedingly likely that the ED clinician will encounter ocular Medium: Print and online trauma on at least a weekly basis. Patients are often understandably Method of participation: Print or online answer form and evaluation anxious about their vision and the threat of vision loss. A system- Prior to beginning this activity, see “Physician CME atic approach to evaluating the injured eye is essential to identify Information” on back page.

Editor-in-Chief Nicholas Genes, MD, PhD Keith A. Marill, MD Scott Silvers, MD, FACEP Research Editor Andy Jagoda, MD, FACEP Instructor, Department of Emergency Assistant Professor, Department of Chair, Department of Emergency Medicine, Mount Sinai School of Emergency Medicine, Massachusetts Medicine, Mayo Clinic, Jacksonville, Lisa Jacobson, MD Professor and Chair, Department Chief Resident, Mount Sinai School of Emergency Medicine, Mount Medicine, New York, NY General Hospital, Harvard Medical FL School, Boston, MA of Medicine, Emergency Medicine Sinai School of Medicine; Medical Michael A. Gibbs, MD, FACEP Corey M. Slovis, MD, FACP, FACEP Residency, New York, NY Director, Mount Sinai Hospital, New Charles V. Pollack, Jr., MA, MD, Chief, Department of Emergency Professor and Chair, Department International Editors York, NY Medicine, Maine Medical Center, FACEP of Emergency Medicine, Vanderbilt Editorial Board Portland, ME Chairman, Department of Emergency University Medical Center; Medical Peter Cameron, MD Medicine, Pennsylvania Hospital, William J. Brady, MD Steven A. Godwin, MD, FACEP Director, Nashville Fire Department Chair, Emergency Medicine, University of Pennsylvania Health Professor of Emergency Medicine Associate Professor, Associate Chair and International Airport, Nashville, Monash University; Alfred Hospital, System, Philadelphia, PA and Internal Medicine, Vice Chair and Chief of Service, Department TN Melbourne, Australia of Emergency Medicine, University of Emergency Medicine, Assistant Michael S. Radeos, MD, MPH Jenny Walker, MD, MPH, MSW Giorgio Carbone, MD of Virginia School of Medicine, Dean, Simulation Education, Assistant Professor of Emergency Assistant Professor; Division Chief, Chief, Department of Emergency Charlottesville, VA University of Florida COM- Medicine, Weill Medical College of Family Medicine, Department Medicine Ospedale Gradenigo, Jacksonville, Jacksonville, FL Cornell University; Department of Peter DeBlieux, MD of Community and Preventive Torino, Italy Emergency Medicine, New York Professor of Clinical Medicine, LSU Gregory L. Henry, MD, FACEP Medicine, Mount Sinai Medical Hospital Queens, Flushing, NY Amin Antoine Kazzi, MD, FAAEM Health Science Center; Director CEO, Medical Practice Risk Center, New York, NY Associate Professor and Vice Chair, of Emergency Medicine Services, Assessment, Inc.; Clinical Professor Robert L. Rogers, MD, FACEP, Ron M. Walls, MD Department of Emergency Medicine, University Hospital, New Orleans, LA of Emergency Medicine, University FAAEM, FACP Professor and Chair, Department of University of California, Irvine; of Michigan, Ann Arbor, MI Assistant Professor of Emergency Wyatt W. Decker, MD Emergency Medicine, Brigham and American University, Beirut, Lebanon Medicine, The University of Professor of Emergency Medicine, John M. Howell, MD, FACEP Women’s Hospital, Harvard Medical Maryland School of Medicine, Hugo Peralta, MD Mayo Clinic College of Medicine, Clinical Professor of Emergency School, Boston, MA Baltimore, MD Chair of Emergency Services, Medicine, George Washington Rochester, MN Scott Weingart, MD, FACEP Hospital Italiano, Buenos Aires, University, Washington, DC; Director Alfred Sacchetti, MD, FACEP Francis M. Fesmire, MD, FACEP Assistant Professor of Emergency Argentina of Academic Affairs, Best Practices, Assistant Clinical Professor, Director, Heart-Stroke Center, Medicine, Mount Sinai School of Inc, Inova Fairfax Hospital, Falls Department of Emergency Medicine, Maarten Simons, MD, PhD Erlanger Medical Center; Assistant Medicine; Director of Emergency Church, VA Thomas Jefferson University, Emergency Medicine Residency Professor, UT College of Medicine, Critical Care, Elmhurst Hospital Philadelphia, PA Center, New York, NY Director, OLVG Hospital, Amsterdam, Chattanooga, TN The Netherlands

Accreditation: EB Medicine is accredited by the ACCME to provide continuing medical education for . Faculty Disclosure: Dr. Alteveer, Dr. Lahmann, Dr. Sharma, Dr. Walther, and their related parties report no significant financial interest or other relationship with the manufacturer(s) of any commercial product(s) discussed in this educational presentation. Commercial Support: This issue of Emergency Medicine Practice did not receive any commercial support. all injuries, to ensure good communications with 1988 the United States Registry (USEIR) consultants, and to instill confidence in the patient. was established to collect information from EDs This issue of Emergency Medicine Practice provides as well as from physicians’ offices; the forms can an up-to-date review of the appropriate evaluation be found online (www.useironline.org) in single- of adults with blunt and penetrating ocular and peri- page format, with a 6-month follow-up report.3 The orbital injuries and offers evidenced-based manage- USEIR also introduced a standard classification ment recommendations. called the Birmingham Eye Trauma Terminology (BETT) to improve data collection and facilitate the Critical Appraisal Of The Literature sharing of information.12 (See Figure 1.)

A literature search was conducted using Anatomy Of The Eye PubMed (www.pubmed.gov), Ovid MEDLINE® (www.ovid.com), University of Medicine and The frontal, maxillary, zygomatic, and lacrimal Dentistry of New Jersey (UMDNJ) electronic books bones come together to form the bony . The and journals, the Cochrane Database of Systematic walls of the orbit are referred to according to their Reviews, and specialty practice guidelines. The anatomic location: superior, inferior, medial, and search was limited to English language literature lateral. The , lacrimal gland, and canalicular published between 1999 and 2009. Search terms system make up the adnexal structures of the eye. included and canalicular lacerations, intraocular (See Figure 2.) The is divided into 2 segments: foreign bodies, corneal abrasions and lacerations, trau- anterior and posterior. (See Figure 3.) matic , and , , orbital The anterior segment includes the , limbal wall fractures, , and . , iris, anterior chamber, and . The From this initial search, more than 120 articles conjunctiva is a thin, transparent mucous membrane were chosen for review, and additional articles that covers the sclera (bulbar conjunctiva) and the were identified through a manual search of the bib- inner surface of the eyelids (palpebral conjunctiva). liographies. High-quality research related to ocular The sclera is a tough layer of collagen and elastic trauma is scarce, and most of the literature consists fiber that surrounds the entire globe, except for the of case series and retrospective analyses. In addi- cornea, and gives the eye its white appearance. The tion, data on eye injuries in the US have been lim- cornea is the anterior-most aspect of the eye. It is ited by the lack of consistency in terminology and transparent, allowing light to be transmitted and ambiguity in the descriptions of these injuries. In focused through the . The iris is a diaphragm

Figure 1. BETT Eye Injury Classification

Eye injury

Open-globe injury Closed-globe injury Full-thickness wound of eyewall Partial-thickness wound of eyewall, ie, sclera and cornea

Laceration Rupture Lamellar laceration Contusion Full-thickness wound of eye- Full-thickness wound of eye- Partial-thickness wound of Blow to eyewall causing wall by sharp object wall by blunt object eyewall partial-thickness wound

Penetrating Perforation Intraocular injury Entrance + exit foreign body Entrance wound wounds only

Note: Some injuries are difficult to define, such as a BB pellet within the vitreous; technically, this is an intraocular foreign body (IOFB) caused by a blunt object. As a result, it can also be described as a “mixed” rupture with IOFB.12 This figure was published in Clinics of North America, Volume 15, Kuhn F, Morris R, Witherspoon CD. Birmingham eye trauma terminol- ogy (BETT): terminology and classification of mechanical eye injuries, pages 139-143, copyright Elsevier, 2002. Used with permission.

Emergency Medicine Practice © 2010 2 EBMedicine.net • May 2010 anterior to the crystalline lens and gives the eye Chemical exposures are the second most its color. The pupil is the circular aperture in the common events.6 In industrialized countries, eye iris that controls the amount of light entering the injuries due to traffic “accidents” are increasing in eye. The size of the pupil is controlled by 2 sets of incidence; just walking along a highway is associat- muscles to the iris: the constrictor muscles (innervat- ed with risk for corneal injury and an IOFB.7 In the ed by parasympathetic fibers from the third cranial US, injuries related to firearms and BB guns repre- nerve) and the dilator muscles (innervated by the sent 12% of injuries. States that allow the private sympathetic nerve fibers). The , located use of fireworks report a much higher incidence of posterior to the iris, produces the aqueous humor. eye injuries than do states where it is prohibited. This transparent, protein-free liquid is contained Sports injuries, especially among baseball and bas- in the anterior and posterior chambers of the eye ketball players, represent 12% of eye injuries. Sixty and provides oxygen and nutrients to the avascu- percent of paintball pellet injuries involve the eye.8 lar cornea and lens. A trabecular meshwork filters When injuries related to eyeglasses are considered and removes the aqueous humor. Contraction and as a separate category, 89% occur in wearers over relaxation of the ciliary body adjusts the thickness of 65 years of age.9 the lens, allowing visual and aiding distance vision. Injury Prevention The posterior segment of the globe contains the ED clinicians have been leaders in advocating for vitreous humor, , , and . mandatory seatbelts, child restraints, and motorcycle The vitreous humor is a clear hydrogel that fills the and bicycle helmets. Public health and professional vitreous cavity. The choroid is the vascular struc- organization websites provide information for edu- ture between the sclera and the retina’s pigmented cating people about appropriate eye protection at epithelium. The optic nerve is a bundle of myeli- work and at play. nated nerve fibers that exits the orbit through the optic foramen. Worksite Safety In the US, the Occupational Safety and Health Epidemiology Administration (OSHA) provides oversight to regulate and enforce protective eyewear on the Eighty percent of patients with significant eye inju- job.4 The National Institute for Occupational ries are male, with an average age of 33 years. The Safety and Health (NIOSH) offers an Eye Safety workplace is the most common site of eye injuries: in Checklist that is available at no charge at their 2004, there were over 36,000 occupational injuries or website (www.cdc.gov/niosh). illnesses involving the eye.4 Typically, foreign matter strikes or is rubbed into the eye. Workers in produc- Highway Safety tion industries, such as welders, cutters, solderers, Public safety initiatives requiring seat belts and and braziers, are most commonly affected4; however, airbags have been shown to save lives and reduce workers in the fields of transportation, farming, min- injuries.10 Laminated windshields – which shat- ing, and construction are also at significant risk.5

Figure 3. Sagittal Section Of The Globe Figure 2. Adnexal Structures

Lacrimal gland Superior punctum Superior canaliculus Caruncle

Medial canthus Nasolacrimal sac Nasolacrimal duct Inferior punctum Inferior canaliculus Used with permission, National Cancer Institute. Common canaliculus

May 2010 • EBMedicine.net 3 Emergency Medicine Practice © 2010 ter into many smaller pieces rather than into large, History 3 sharp shards – have also helped reduce eye injuries. The EMT or the ED clinician should attempt to determine how, when, and where the eye injury oc- Protective Eyewear In Sports curred and what the injury entails, making sure that Over the past 20 years, the use of protective eyewear the following information is requested. by athletes who participate in ice hockey (especially • Mechanism of injury: How did the injury oc- in youth leagues), racquetball, and squash has had cur? The answer elicited should be as detailed as a positive impact on the incidence of serious eye possi­ble. Ask the patient or witness to describe 3 trauma. In 1995–1996, the American Academy of the event and ask whether the injury was blunt Ophthalmology, together with the American Acad- or penetrating, whether an airbag deployed, and emy of Pediatrics, published a joint policy statement whether protec­tive eyewear was in place. If the on the use of protective eyewear during sports. The injury is an IFOB, remember that 20% of all such guidelines include specific recommendations for a patients have no as­sociated pain. variety of sports and the minimum eyewear re- • Time of injury: When did the injury occur? Is quirements for each type of sport based on research this a recent (“fresh”) penetrating injury that carried out by the American Society for Testing and might need emergency surgery, or did the injury Materials (ASTM). Updated and reapproved in 2003, occur a few days ago? A delayed presentation these guidelines were designed specifically for the that appears “qui­et” (ie, with no evidence of protection of young athletes, but they can be ex- infection) might continue to be managed expec- trapolated for anyone engaging in a sport that puts tantly and nonoperatively. the unprotected eye at increased risk for a traumatic • Place of injury: Where did the injury occur? Was 11 injury. (See Table 1.) it in a rural environment? • Composition of IOFB: What might the foreign Prehospital Care body contain? Is it organic material (vegetal) or metallic? If the latter, what was the specific type There are no evidence-based guidelines for the of metal? If the object is composed of chemicals, prehospital management of eye injuries. Local is it basic or acidic? Is it a biologic or chemical protocols may vary depending on the emergency weapon? Answers to these questions are critical medical services (EMS) command, trauma triage both for determining the method of evaluation guidelines, the number and proximity of receiv- and for optimal management. ing hospitals, and the length of transport time. In • Past medical history: Are there any general all cases, however, the ABCs of resuscitation and medi­cal conditions or medications that could trauma evaluation take precedence over the man- affect the visual outcome? For instance, diabetes agement of ocular injuries. and chronic steroid use are well-known risk fac- Textbooks for emergency medical technicians tors for poor healing. (EMTs) and paramedics provide a general approach • Previous ocular surgery: Had the patient under­ to eye injuries that covers 3 discrete areas: (1) decon- gone any eye surgery in the past? The site of a tamination of chemical exposure; (2) stabilization previ­ous surgical incision is likely to be a weak of a protruding object; and (3) protection of the eye point, predisposing to an open-globe injury. from further damage. Any object protruding from • Current ocular medications: The use of intraoc- the eye should not be removed but should instead ular steroids prior to the injury may predispose be stabilized during transport.13 If there is nothing to infec­tion. obviously sticking out of the eye and the patient can- • Baseline vision: What was the patient’s vi- not open the eye, the EMS provider should refrain sion like before the accident? It is important from forcibly opening the lids and should apply to establish visual acuity both before and after either a metal or a rigid plastic shield over the eye. the injury in each eye and in both . Injury If such a shield is not available, a disposable cup can to the good eye carries great significance for a be taped over the injured eye.14 patient who is functionally or physically blind in the other eye.14 ED Evaluation Physical Examination The goal of the “ophthalmologic” evaluation is a The key to examining the injured eye is to use a systematic and logical delineation of any and all systematic approach while remaining vigilant for injuries. Life- and limb-threatening conditions take possible penetration or rupture of the globe. First, do priority. For example, in the setting of multiple trau- an external inspection, followed by an assessment matic injuries, the evaluation of the ruptured globe of visual acuity, central and peripheral vision, and may need to be deferred until other major systemic ocular motility. Next, if possible, evaluate the spe- or brain injuries are addressed. cific areas of the eye from “outside in” using a slit

Emergency Medicine Practice © 2010 4 EBMedicine.net • May 2010 lamp; if the patient is unable to sit still or cooperate, ing the line at which 50% of the letters are correctly a handheld light and magnifying glass may be used identified (eg, 20/100). instead. Finally, perform direct ophthalmoscopy, but For many moderately or severely injured pa- remember that dilation is contraindicated in patients tients, standing up and reading a chart is not feasible with an open-globe injury. or practical. The next best option is a near card, held 14 inches from the eye(s). With this method, 14 External Examination becomes the numerator, and the line at which 50% Under a bright light, inspect the scalp, face, perior- of the letters can be identified is the denominator. If bital tissues, and eyelids for lacerations, ecchymoses, the patient’s eyeglasses are lost or missing, a pinhole foreign bodies, and edema. Inspect the globe for card may act as a substitute for them. Several small protruding foreign bodies, prolapse of ocular con- pinholes are made in a piece of cardboard and held tents, hemorrhage, irregular , and pronounced in front of the patient’s eyes, 1 at a time, as vision is . Stabilize, but do not remove, any again checked with the Snellen chart or near card. protruding foreign body until controlled removal The pinhole card allows parallel rays of light to pass can be accomplished in the operating room (OR). through the holes and land directly on the retina If you suspect globe rupture, stop the examination, without the need for refraction (ie, ). When a avoid placing any pressure on the orbit, and lightly patient who usually wears glasses is unable to see tape a protective shield over the eye. Contact the with this method, this is suggestive of a pathologic ophthalmologist on call, who will need to do a full lesion interfering with visual acuity.15 examination in the operating room under controlled For patients who are unable to see the letters, circumstances. Only after you are certain that there the ED clinician can employ other means of assess- is no globe rupture or penetration, apply gentle ment. In decreasing order of their abilities, patients traction to the eyelids or gently use a lid retractor or should be asked to count fingers, to detect gross speculum to gain access to the visual axis. hand movements, and finally to respond to light by estimating perception as bright, barely, or none. Visual Acuity Changes in color vision may also reflect optic nerve Ideally, determine visual acuity using a Snellen defects; when the optic nerve is damaged, red ob- chart. Each eye should be evaluated separately, with jects will appear grayish or washed out.14 the injured eye evaluated first to prevent memoriza- tion of letters on the chart (assuming poorer vision Afferent Pupillary Defect in the injured eye).15 Have the patient sit or stand 20 An important indicator of visual function is the feet from the chart; the exact procedure for reading presence or absence of an afferent pupillary defect the eye chart has been well described elsewhere.15,16 (APD), also referred to as a relative afferent pupil- Record the results as a fraction, with the numerator lary defect (RAPD). In the normal eye, the pupil as the distance to the chart and the denominator be- constricts to direct light (the direct light reflex) but

Table 1. Risk Of Ocular Injury According To Type of Sport, With Special ASTM Eyewear Recommendations

High Risk Moderate Risk Low Risk Safe Gun sports: Tennis Swimming Track and field Air gun Badminton Diving (Javelin and discus require good field supervision) BB gun Soccer Skiing (snow or water) Gymnastics Paintball (ASTM 1776) Volleyball (ASTM F659) Balls and “sticks”: Water polo Wrestling Baseball (youth) (ASTM F910) Football Bicycling Softball Fishing Martial arts — no contact Cricket Golf Ice hockey (ASTM F513) Field hockey Racquet sports with close contact: Squash Racquetball Fencing Boxing Martial arts — full contact

American Society for Testing and Materials (ASTM) recommendations should be used when purchasing or using protective eyewear. Unless otherwise noted, the eyewear recommended by the ASTM for all sports is ASTM F803.

May 2010 • EBMedicine.net 5 Emergency Medicine Practice © 2010 also when light is shone into the other eye (the the patient has (or is suspected of having) an open- consensual reflex). This requires intact iris muscles, globe injury, this test should not be performed! The retina, and optic nerve. The swinging flashlight test patient must be able to sit up and cooperate. Before to detect an APD is performed as in Figure 4.15,16,17 examining the conjunctiva and sclera, administer a A penlight or other light source is shone first in one few drops of (such as 0.5% propa- eye and then in the other, swinging back and forth. racaine or 0.5% tetracaine), but be sure to use an un- Both pupils should constrict when light is shone opened bottle to prevent the introduction of possibly in either eye and dilate when the light source is contaminated material into the injured eye.14 removed (as in the left column). If, as in the center To evert the eyelid, ask the patient to look down, column, the left eye has an afferent defect due to and applying a clean, cotton-tipped swab to the damage to the retina or optic nerve, the left eye will superior palpebral sulcus, grasp the with constrict to light shone in the right eye but dilate to your free hand and gently lift the lid back over the direct light (preserved consensual reflex). The term swab. Do a thorough exploration and eversion of “relative” is used when the pupil constricts a little both lids to look for foreign bodies that might be but less than the normal eye. The right-hand column sequestered under the lids. Again, do not perform indicates what happens if the left pupil has both an eversion if there is an obvious open-globe injury. In afferent as well as an efferent deficit. gathering data, remember that conjunctival lacera- tions may indicate underlying injury to the sclera Visual Fields And Ocular Motility and that hemorrhagic chemosis may indicate orbital At the bedside, conduct gross, confrontational fracture or open-globe injury.14 visual-field examinations of each eye separately. To The Seidel test is used to detect full-thickness assess ocular motility, have the patient look in all 4 corneal laceration or rupture. This test is performed directions while keeping the head pointed straight using fluorescein dye and a cobalt-blue light. A ahead. Entrapment of ocular muscles (eg, with an few drops of 2% fluorescein dye are placed on the orbital wall fracture) can be demonstrated by asking cornea, and the result is positive when aqueous fluid the patient to look upward. However, do not per- leaking from a corneal laceration appears as a green form this test if globe rupture is suspected.14 stream in the middle of a pool of bright yellow.14,15 Next, the anterior chamber should be examined Slit-Lamp Examination for the presence of abnormal cells. Any inflamma- The slit lamp is used to examine the conjunctiva, tory cells will appear similar to dust specks floating cornea, sclera, anterior chamber, iris, and lens. If in a brightly sunlit room; red blood cells appear as brown dust.15 Blood in the anterior chamber will appear as a brown or red meniscus in front of the Figure 4. Swinging Flashlight Test iris, called a hyphema. This type of hemorrhage can often be seen with the naked eye or a magnifying Normal APD APD with a fixed and lens. (See section on Hyphema on page 10.) Ideally, dilated pupil the ED clinician should also try to assess the depth

Normal Eye Abnormal Eye Normal Eye Abnormal Eye of the anterior chamber. A “deep” chamber may in- dicate posterior dislocation of the lens, , or scleral rupture, whereas a “shallow” chamber can occur with anterior lens dislocation, vitreous pro- lapse, leaking corneal or scleral wounds, choroidal 14 Normal Eye Abnormal Eye Normal Eye Abnormal Eye injury, or closed-angle glaucoma. Last, examine the crystalline lens for dislocation or cataract formation. In the absence of a slit lamp, a handheld light and ophthalmoscope set at +6 to focus on the cornea and anterior chamber should Normal Eye Abnormal Eye Normal Eye Abnormal Eye facilitate identification of a hyphema, a cataract, or pupillary irregularities.

Intraocular Pressure Measurement In open-globe injuries, measurement of the intraocu-

Normal Eye Abnormal Eye Normal Eye Abnormal Eye lar pressure (IOP) should be deferred until after sur- gical repair has been accomplished. Obtain pressures using a Schiotz tonometer, a hand-held tonometer, or a Goldmann applanation tonometer attached to the slit lamp.15 By consensus, normal IOP is considered APD APD to be between 10 and 20 mm Hg. Low pressures may Abbreviation: APD, afferent pupillary defect. be an indication of occult globe rupture or lacera-

Emergency Medicine Practice © 2010 6 EBMedicine.net • May 2010 tion, injury to the ciliary body, or . slices may be needed to localize IOFBs. CT is also in- A normal or even high IOP does not rule out open- dicated when the posterior segment cannot be visu- globe injury or rupture. IOP may be elevated imme- alized and in cases of suspected occult globe rupture diately after contusion to the globe, in the presence or laceration and with metallic foreign bodies.14,20 of cells in the anterior chamber, in mechanical angle For the past 20 years, magnetic resonance imag- closure, and with anterior dislocation of the lens.14 ing (MRI) has been used in the diagnosis of orbital and periorbital tumors and to characterize optic Direct Ophthalmoscopy nerve disorders.21 Unlike US, MRI can be used to Examination of the posterior segment (vitreous, delineate small, wooden, and organic matter foreign retina, and optic nerve) performed early in the ED bodies; however, the operator must be sure that no visit is probably the best opportunity to see the metallic IOFBs are present, since application of the fundus before the “view” is obscured by hyphema, magnetic field risks further damage from shifting of traumatic cataract, or .14 A 0.5% the foreign object.7 Disadvantages of MRI include ophthalmic solution of tropicamide, a short-acting the relatively long scanning times and the need for papillary dilator, may be useful for a better fundo- patient cooperation to prevent motion artifact.21 scopic view. Again, ophthalmoscopy is contraindicated in Decision Models For Predicting Visual patients with open-globe injuries. In cases of severe injury or open-globe rupture, the posterior segment Outcome After Injury may be evaluated by computed tomography (CT) or ultrasound (US). If neither of these modalities is avail- The ocular trauma score (OTS) was developed in able, ophthalmoscopy may need to be deferred until a 2002 by Kuhn et al using data from the USEIR and ruptured globe or lacerations have been repaired. the Hungarian Eye Injury Registry and is used to predict visual outcome after injury.22 Points are as- signed for several variables, including initial vision, Imaging In Ocular Trauma presence of rupture, endophthalmitis, perforation, retinal detachment, and RAPD, and the totals fall Ultrasonography and CT scanning are the 2 most into 5 prognostic categories, with higher scores re- useful modalities in evaluating severe ocular trauma. flecting better initial visual acuity and fewer struc- With use of a high-frequency probe (7.5-10 MHz), tural defects. In general, the higher the initial score, US allows the ED clinician to rapidly evaluate many the better the prognosis for a good visual outcome. intraorbital structures. It can be used to delineate These categories can be useful in preparing the choroidal and scleral lacerations, vitreous hemor- rhages, retinal detachment, radiolucent and radio- opaque foreign bodies, and retrobulbar hematomas (See Figure 5.). Ultrasound can be performed at Figure 5. Ultrasound Image Of Detached the bedside within a few minutes of the patient’s Retina arrival in the ED. A small prospective observational trial of bedside US performed by ED physicians in traumatic and nontraumatic eye conditions demon- strated a sensitivity of 100% and a specificity of 97% compared to confirmatory CT or ophthalmologic examination.18 Ultrasound is contraindicated when the risk of extrusion of intraocular contents is high, such as with large globe lacerations or the uncooperative patient.14 In addition, this modality may miss small, wooden, or organic foreign bodies, and the pres- ence of small gas bubbles may yield false-positive results.7 In the cooperative patient, a large amount of ultrasonic gel can be applied to the closed lid, thus cushioning the linear transducer and avoiding its direct contact with the eyelid or globe. The eyelids remain closed and the patient will experience mini- mal additional discomfort.18,19 CT scans are currently the most common modal- ity used in hospitals to evaluate the injured eye. Both axial and coronal views should be ordered to detect any orbital fractures. Ultrathin (1 mm) tomographic Used with permission from Bret Nelson, MD.

May 2010 • EBMedicine.net 7 Emergency Medicine Practice © 2010 patient and family for a potentially poor outcome light, the entire cornea is examined. Abrasions will as well as for ophthalmology decision-making, appear yellow-green. research, and public health interventions.16 The OTS Minor corneal abrasions are treated with topi- was recently validated in a retrospective review cal or oral nonsteroidal anti-inflammatory drugs of 114 patients who presented to 1 institution over (NSAIDs) and topical antibiotics. Common topi- a period of 5 years, finding a positive correlation cal NSAIDs used include diclofenac and ketorolac. between the OTS and the patients’ ultimate visual Studies have shown that topical NSAIDs are effec- acuity (P < 0.001).23 tive at reducing pain, allowing lower doses of oral In 2008, Schmidt et al published an outcome analgesics and narcotics, and allowing the patient to classification and regression tree (CART) based on return to work sooner.27 In clinical trials conducted retrospective analysis of a cohort of patients with more than 10 years ago outside the US, topical open-globe injuries evaluated at a single institu- antibiotics not commonly used in this country (ie, tion.24 The study was set up using the CART analy- chloramphenicol) produced conflicting results sis, with a separate cohort to test the sensitivity and with regard to their benefit. To date, no new data specificity of the analysis. The most important factor have been reported. If a topical antibiotic is used, in predicting poor outcome was the presence of it should be affordable and safe and be associated RAPD: 62.7% of patients with RAPD had no vision, with a low incidence of reactions versus 3% when RAPD was not present. The second (eg, polymyxin B/trimethoprim ophthalmic drops). strongest predictor was level of initial vision.24 Neomycin preparations are associated with an Man and Steel recently compared the OTS with unacceptably high incidence of sensitivity reactions. the CART model.25 In a retrospective analysis of 100 Most corneal abrasions heal within 24 to 72 hours, consecutive open-globe injuries at a single institu- and outpatient follow-up with an ophthalmologist is tion, they compared CART and OTS predictions usually necessary only for those patients with deep with actual visual outcomes and calculated the sen- or large abrasions or those whose symptoms persist sitivity and specificity of each model. The variables for longer than 48 hours.28 most closely predictive of poor visual outcome were A number of studies from the 1980s and early RAPD, poor initial vision, lid laceration, posterior 1990s documented an association between soft wound, and globe rupture. Both models were highly contact lens use and susceptibility to Pseudomonas sensitive in predicting light perception or better vi- colonization and infection,29 since these lenses have sion (97.4% for OTS and 93.5% for CART). However, been shown to enhance pseudomonal adherence to the OTS was 100% specific in predicting no vision the cornea.30 No randomized, prospective trials have (vs 73.9% with CART) and very poor vision (vs been conducted to confirm this. If a patient with a 81.8% with CART).25 wears contact lenses, or if the cor- For the ED clinician, the take-home point is that neal abrasion was the result of contact with vegetal poor initial vision and RAPD are associated with matter (eg, from a piece of wood), antibiotic cover- worse outcomes after ocular trauma. age should include antipseudomonal activity (eg, ciprofloxacin or ofloxacin ophthalmic drops). Such Blunt Eye Trauma patients should return for a follow-up visit with an ophthalmologist within 24 hours even if they are Corneal Abrasions feeling well. Contact lenses should not be worn until the abrasion has healed and the course of antibiotic Abrasions to the cornea are very common and may 27 potentially lead to long-term sequelae. The cornea therapy has been completed. is the thin, transparent outermost covering of the A 2006 Cochrane review on eye patching after eye. It is made up of multiple layers and provides a corneal abrasion identified 11 trials with a total of physical barrier to the eye, protecting it from infec- 1014 patients. These studies looked at pain reduction tion and trauma. In terms of history, the patient and rate of healing and concluded that minor corneal may or may not remember a traumatic event.26 abrasions do not benefit from eye patching. Moreover, Presentations usually include acute pain, tearing, use of an eye patch causes monocular vision and may , sensation of a foreign body, blurry present a safety risk. Based on good-quality evidence, eye patches are no longer recommended for minor vision in the affected eye, and possibly headache. 31 These symptoms usually worsen with exposure to corneal abrasions less than 10 mm. light, repeated blinking, or rubbing of the eye. Direct Complications from simple corneal abrasions visualization of the cornea, preferably using a slit are rare; however, a syndrome of recurrent corneal lamp, is necessary to make the diagnosis. A topical erosion has been described that presents as repeated anesthetic such as proparacine or tetracaine should spontaneous corneal epithelial defects that may oc- be instilled in the eye to facilitate the examination, cur on awakening or after rubbing the eyes. Symp- followed by the application of fluorescein dye. The toms are similar to corneal abrasions but milder. The room is darkened and using a cobalt-blue filtered lesions appear at or near the original area of injury and are thought to be due to weakened corneal

Emergency Medicine Practice © 2010 8 EBMedicine.net • May 2010 tissue.27 Treatment is similar to that for the initial term that refuses to go away and may still be used abrasion, with referral to an ophthalmologist in in day-to-day practice. In traumatic iridocyclitis, cases of persistent symptoms or deep lesions.32 Pro- the iris and its attachment to the anterior ciliary phylactic measures such as topical lubricating gels body become irritated and inflamed, resulting in or topical antibiotics for a period of 2 months are and spasm of accommodation. The annual generally acceptable, although the evidence for these incidence of uveitis is 17 in 100,000 people. Ante- measures is slim.31 According to a recent Cochrane rior uveitis is more common than posterior uveitis Review, evidence was not sufficient to support other and usually occurs between the ages of 20 and 50 therapies for refractory cases, such as contact lenses but rarely before age 10 or after age 70. versus oral tetracyclines versus laser or surgical On presentation, the patient with uveitis may treatments.33 (See Table 2.) complain of a painful, with tearing, photo- phobia, and blurry vision. The conjunctiva of the Traumatic Uveitis (Iridocyclitis), Traumatic affected eye may be injected, with a small, poorly , And Traumatic Iridodialysis dilating pupil. Light directed at the unaffected eye The is composed of the iris, ciliary body, and will result in pain and photophobia in the affected choroid. These structures are usually flexible enough eye. The presence of cells and flare in the anterior to withstand moderate trauma without much dam- chamber on slit-lamp examination, along with age; however, with more severe blunt trauma, injury miosis and pain, are diagnostic of anterior uveitis. (either structural damage or a post-traumatic inflam- Intraocular pressure should be measured to identify matory response) can occur immediately or several a more serious injury to the globe, such as secondary days later.34 glaucoma. The disease process is self-limited and usually resolves within 7 to 14 days. Although there Traumatic Uveitis are no outcomes data from randomized, controlled The iris is the most anterior portion of the uveal trials to support the current treatment for traumatic 35 tract. Inflammation of this portion of the uvea has uveitis, recommendations include cycloplegic been saddled with changing terminology over agents, prednisolone acetate 1%, and discharge the years. “Uveitis” or “anterior uveitis” are the home, with outpatient follow-up with an ophthal- common terms and are often used interchange- mologist within 2 to 3 days. Cycloplegic agents such ably. “Iridocyclitis” describes the condition more as homatropine 5% for 7 to 10 days will relieve the specifically, referring to the part of the uvea that pain associated with ciliary spasm and may pre- is actually inflamed, but it is not often used in the vent adhesions from developing between the pupil emergency medicine literature. “Iritis” is an old and lens. Prednisolone acetate 1% achieves a high concentration in the anterior chamber and is thought

Table 2. Treatment Of Traumatic Ocular Emergencies

Condition Treatment Clinical Pearl Corneal abrasion • NSAIDs, either topical or oral Think of Pseudomonas exposure with contact lens users • Topical antibiotics and exposure to vegetal material • Oral narcotics Traumatic uveitis (irido- • Topical cycloplegics, eg, homatropine 5% Measure IOP to detect associated injuries such as second- cyclitis) • Topical steroids, eg, prednisolone acetate 1% ary glaucoma • Ophthalmology consult with follow-up in 2-3 days Traumatic glaucoma Treat with all: Miotics may be contraindicated in patients who have under- • Topical α-blocker gone cataract surgery or lens extraction • Topical β-blocker • Carbonic anhydrase inhibitor (acetazolamide 500 mg IV or PO) • Hyperosmolar agent (mannitol 1 to 2 g/kg IV) • Re-evaluate IOP hourly • When IOP < 40 mm Hg, start topical cholinergic agent (ie, pilocarpine) • Ophthalmology consult Hyphema • Rigid shield to protect eye Check IOP: elevated IOP is an indication for admission; • Elevate head of bed 30° prolonged elevated IOP is an indication for surgery • Oral analgesics • Topical cycloplegics • Topical steroids

Abbreviations: IOP, intraocular pressure; IV, Intravenous; PO, by mouth.

May 2010 • EBMedicine.net 9 Emergency Medicine Practice © 2010 to reduce the inflammatory response. In severe Elevated IOP should be treated aggressively, and cases, it may be administered hourly initially and pressures exceeding 21 mm Hg should prompt a call then gradually reduced.35 Complications of uveitis to the ophthalmologist.41 include cataract formation and glaucoma; however, The treatment of traumatic glaucoma is similar since steroid treatment itself has been associated to that of acute narrow-angle glaucoma. In the ED, with these 2 complications, prednisolone therapy the patient should be given a topical b-blocker, a should be initiated in consultation with an ophthal- topical a-blocker, a carbonic anhydrase inhibitor mologist, with the assurance of follow-up visits. (See (acetazolamide 500 mg intravenously [IV] or orally), Table 2, page 9.) and a hyperosmolar agent (mannitol 1 to 2 g/kg IV). Intraocular pressure should be reevaluated hourly. Traumatic Mydriasis And Miosis Once IOP drops below 40 mm Hg, a topical cholin- Traumatic mydriasis (pupillary dilatation) occurs ergic agent such as pilocarpine should be started. when there are small tears to the sphincter muscle of Miotics may be contraindicated in patients who the iris.36 Traumatic miosis (pupillary constriction) have previously undergone cataract surgery or lens occurs when the iris sphincter becomes damaged extraction.42 (See Table 2, page 9.) and irritated. With either entity, the patient often A 2007 Cochrane review that included 26 trials complains of eye pain or blurry vision. In a patient with 4979 participants clearly demonstrated medi- with significant head trauma or altered mental cal management to lower IOP offers a protective status, the possibility of cranial nerve palsy due to effect on the visual field.41,43 Although these studies increased intracranial pressure must be ruled out do not include traumatic glaucoma, ophthalmolo- first by means of appropriate neuroimaging.37 For gists believe that this benefit applies to glaucoma of isolated traumatic mydriasis or miosis, no specific any etiology. ED treatment is indicated since the condition often resolves spontaneously.36 Hyphema Hyphema is defined as blood in the anterior cham- Traumatic Iridodialysis ber. This may occur in one-third of cases of serious In traumatic iridodialysis, the ciliary body is torn ocular trauma. The estimated annual incidence from the iris root, leading to the formation of a “sec- of hyphema is 17 to 20 per 100,000 population, ondary pupil.” The patient may complain of pain as with most patients being younger than 20 years of well as double vision in the affected eye, but visual age.44 The sources of bleeding are the blood vessels acuity should be unaffected. Because of the force re- in the iris or ciliary body. Hyphemas are classified quired to tear the ciliary body, a full from grade 0 to 4, based on the percentage of the must be done. If there is a hyphema or visual acuity anterior chamber filled with blood. Grade 0 repre- is decreased, retinal or vitreous injury should be sus- sents microhyphemas in which circulating red blood pected and an urgent ophthalmologic consultation is cells can be detected only by slit-lamp examina- warranted. If there is persistent monocular , tion. The grading system then progresses from grade surgical repair may be indicated.36,38 1 (less than one-third of the anterior chamber) to grade 4 (anterior chamber filled with blood).44 (See Traumatic Glaucoma Figure 6.) The term “eight-ball hyphema” refers to Blunt or may cause acute trau- an anterior chamber that is entirely filled with blood, matic glaucoma. The increase in IOP may occur ear- making it appear like a black ball.36 Symptoms of ly or late after the injury. Although it can be due to hyphema include pain, photophobia, and blurry various mechanisms, the increased pressure is usu- vision. With a grade 4 hyphema, there is sudden ally due to impaired trabecular drainage.39 In a re- vision loss, high IOP, extreme pain, and nausea.45 cent cohort study from the USEIR published in 2005, Lethargy or somnolence can be associated with iso- the risk of developing post-traumatic glaucoma was lated traumatic hyphema but should raise concern very low (2.67%).40 It is more likely to occur after a for a concomitant . Traumatic miosis or closed-globe injury (77%) than an open-globe injury mydriasis may be present, but not an APD.36 (23%).38 The development of glaucoma was indepen- The goals of treatment in traumatic hyphema are dently associated with advancing age, lens injury, to prevent repeated eye trauma and rebleeding, to poor baseline visual acuity, and inflammation.40 The promote the settling of blood away from the visual presentation is similar to acute angle closure or axis, and to control anterior uveitis and increases “narrow angle” glaucoma, with patients complain- in IOP. As a result, treatment may run the spectrum ing of cloudy vision and eye ache or pain. Nausea from supportive care and medical management to and vomiting are common. In healthy patients, the surgical intervention. optic nerve can tolerate moderately elevated IOP for Most patients with hyphema can be treated at a short period of time; however, prolonged eleva- home. Moderate activity (such as walking and read- tions in pressure can result in permanent vision loss. ing) is allowed. When the patient is lying down, the

Emergency Medicine Practice © 2010 10 EBMedicine.net • May 2010 head of the bed should be elevated to an angle of nerve damage and corneal blood staining), IOP 30°. A metal shield should be applied over the entire greater than 35 mm Hg for 7 days, total hyphema orbit to protect the eye from further injury until the still unresolved after 9 days, or microscopic corneal hyphema resolves. The shield must have holes or be blood staining.45 Surgery has also been indicated for made of clear plastic so patients can monitor their patients with sickle cell disease or sickle trait.44 Sur- vision, since a decrease in vision is the earliest symp- gical techniques include paracentesis, anterior tom of rebleeding. chamber washout, expression of the clot, automated Medical management may include oral analge- removal of blood, and trabeculectomy.45 The most sics as needed and topical cycloplegics for patient common complication of a traumatic hyphema is comfort. Topical steroids are believed to reduce rebleeding, which occurs 2 to 5 days after the injury intraocular inflammation and reduce the risk of when the initial clot retracts and loosens.36 A hyphe- secondary hemorrhage; however, the evidence is ma that fills 75% of the anterior chamber typically contradictory (see discussion below). Topical and results in a traumatic cataract and is associated with systemic antifibrinolytics such as aminocaproic vitreous hemorrhage in about 50% of cases.45 (See acid (ACA) and tranexamic acid delay clot dissolu- Table 2, page 9.) tion, thereby acting to reduce secondary hemor- rhage.36,43,44 Aspirin and NSAIDs should not be used Lens Injuries: Post-Traumatic Cataract because of an increased risk of rebleeding. Cataract formation is the most common result of Results in the recent literature with regard to injury to the lens, with a 39% incidence in open- the use of steroids in hyphema are conflicting. In a globe injuries and an 11% incidence in closed-globe retrospective cohort study published in 2008, pedi- injuries.44 The traumatic event may be blunt trauma atric patients with hyphema who were treated with (the major cause), exposure to a high-voltage tranexamic acid plus topical steroids did not have electrical current, laser beams directed at the eye, a significantly lower incidence of rebleeding than or a lightning strike. Bilateral can de- those treated with topical steroids alone.46 Corticos- velop after electrical injury and lightning strikes. teroids have been used to treat hyphema and appear Traumatic cataracts may occur acutely or develop to be effective. Topical steroids have been shown to over weeks to months and may therefore be diffi- have fewer adverse side effects, but their effective- cult to diagnose when a patient presents long after ness is unclear. the initial injury. A recent case report describes Hospitalization has been recommended for pa- the development of an electrical cataract as the tients with hyphema who have rebleeding, elevated result of a Taser® gun attack.47 With a lens injury IOP, hyphemas involving more than 50% of the (as well as a unilateral corneal or retinal injury), anterior chamber, or decreased visual acuity, as well monocular diplopia is present. (Binocular diplopia as for noncompliant patients and in suspected cases implies a problem with the or of child abuse.36 Indications for surgical interven- nerves.) Definitive treatment usually involves cata- tion after hyphema include IOP greater than 50 ract removal, thus requiring patient referral to an mm Hg for 5 days (since a prolonged elevation in ophthalmologist. The visual prognosis for patients IOP is associated with an increased chance of optic with traumatic cataract but without additional intraocular trauma is usually excellent.44

Figure 6. Grade 3 Hyphema Trauma To Implanted Lenses After cataract surgery involving removal of the native lens and implantation of a standard polym- ethylmethacrylate , patients can sustain trauma to the implanted lens. Although implanted lenses are durable, are resistant to aging and climate, and can withstand the level of stress necessary for implantation, blunt trauma can cause the lens to fracture and dislocate. This is a very rare event, and the patient presents with pain and blurry or double vision.48 Other injuries involving the native lens are sub- luxation and dislocation injuries. Blunt trauma to the eye can cause damage to the zonule fibers, resulting in dislocation of the lens (posteriorly more com- monly than anteriorly). The patient will complain of blurry vision or monocular diplopia. An anteriorly displaced lens can be visible on direct or slit-lamp Used with permission from Rakesh Ahuja, MD.

May 2010 • EBMedicine.net 11 Emergency Medicine Practice © 2010 examination, whereas the pupil must be dilated to Orbital Fractures view a posteriorly displaced lens. Ultrasound may 18 be useful for locating the lens in this situation. All A direct blow to the eye can cause an orbital wall patients with dislocated lenses, whether native or fracture, also known as an orbital “blowout frac- implanted, should be referred to an ophthalmologist ture.” This term was coined in 1957, when Smith and for surgical repair. Emergent repair is necessary if Regan described the mechanism of injury. The me- 36 there is elevated IOP. dial and inferior walls of the orbit are fractured as a result of increased hydraulic pressure from the force Injuries To The Posterior Segment (Vitreous, of the blow to the globe. The fractures are usually Retina, And Posterior Choroid) confined to the orbital walls and do not reach the Posterior Segment Injury Associated With Anterior orbital rim.54 The patient may present with the clas- Segment Injury sic triad of (recession of the eyeball in The ED clinician should consider the possibility of the socket), restrictive (diplopia on up- occult posterior injury for all patients with high- ward gaze), and infraorbital numbness (anesthesia velocity or high-risk, seemingly “isolated” anterior below the eye along the infraorbital nerve distribu- segment injury. Inflating airbags can cause hypo- tion).55 Other clinical findings may include a step- tonic , retinal detachment, macular hole off deformity on palpation of the infraorbital rim formation, and traumatic .49,50 Vehi- and intraorbital emphysema. In patients with blunt cle occupants 66 years of age and older were found trauma who complain of diplopia, one must also to be 2 to 3 times more likely to incur an eye injury consider a diagnosis of orbital hemorrhage or edema after airbag deployment. In addition, patients who (without blowout), bruised extraocular muscle, and have undergone refractive surgeries such as radial cranial nerve palsy. Plain radiographs are usually keratotomy, photorefractive keratectomy, and LASIK inadequate; CT scans, both axial and coronal, are the are also at increased risk for posterior segment inju- modality of choice. (See Figure 8.) ry.50 Airgun, stun gun, or paintball injuries should be There are no well-designed prospective stud- included in this group of patients at high risk51 and ies on the emergency management of orbital wall all should be followed up by an ophthalmologist. fractures. Based on historical practice, as well as expert opinions in the fields of emergency medicine Retinal Detachment Related To Trauma and ophthalmology, the initial treatment consists of According to the USEIR, the retina is involved in intermittent application of ice or cold to the site for nearly one-third of all serious eye injuries, and the 48 hours, the use of nasal decongestants, elevation retina or vitreous is involved in almost 50% of all of the head of the bed, avoidance of aspirin (unless severe open- or closed-globe injuries.52 In adults, trau- required for other medical reasons), and an injunc- ma accounts for only 10% of all retinal detachment, tion against blowing the nose. In addition, antibiot- whereas in children trauma is the leading cause. ics that cover sinus pathogens, such as high-dose Retinal tears themselves do not cause pain. amoxicillin (875 mg every 12 hours for 7 to 10 days) The patient may complain of seeing flashes of or amoxicillin-clavulanate are often recommended light or sparks, cloudy or smoky vision, or the for patients with severe injury or subcutaneous air. more classic “cloudy curtain” descending over the Immediate surgery is recommended for patients field of vision. On fundoscopy, a hazy gray mem- with large fractures, hypoglobe (downward dis- brane of retina may be seen billowing forward; placement of the eye), significant enophthalmos however, many tears are located in the periphery (posterior displacement of the eye), or signs of and are not seen with direct ophthalmoscopy. entrapment. Delayed surgery (ie, within the first 2 Visual acuity may be normal unless the macula is weeks after injury) is recommended for enophthal- involved.53 Indirect ophthalmoscopy or US may mos or persistent diplopia. Supportive medical be needed to demonstrate the tear.18 (See Fig- management is for those who do not meet criteria ure 5, page 7.) Retinal detachment may also be for surgery.56,57 The choice of consultant — whether seen on CT scan. (See Figure 7.) Sometimes after plastic surgery, ENT, oral, and/or maxillofacial sur- acute blunt trauma, a traumatic macular hole can gery — together with ophthalmology is dependent develop in the posterior segment. The underly- on regional practice patterns. ing mechanism is not well understood but may involve traction by overlying adherent vitreous. Retrobulbar Hemorrhage Retinal tears, macular holes, and vitreal injury The bony orbit is a confined space, with little room require emergent ophthalmologic consultation. for expansion. Retrobulbar hemorrhage is a rare Surgical intervention is needed to repair the detach- complication of nondisplaced orbital fractures. In ment, perform a vitrectomy in the setting of vitreous displaced orbital fractures, the blood escapes into hemorrhage, or repair the macular hole. In most the surrounding sinuses. In nondisplaced fractures, 52 cases today, such repair is successful. the blood accumulates behind the orbit, resulting in

Emergency Medicine Practice © 2010 12 EBMedicine.net • May 2010 a buildup of pressure behind the globe. This results hemostat or Kelly clamp is used to crimp the skin in pressure on the globe and traction on the optic at the lateral canthus for 1 to 2 minutes to minimize nerve, described by some as an ocular compartment bleeding and to mark the site of the canthotomy. The syndrome.16 Patients with retrobulbar hemorrhage canthus is incised, using small scissors to make a usually complain of severe eye pain, reduced eye 1- to 2-cm lateral incision along the line made by movement, and/or a change in vision. the clamps, beginning at the lateral canthus and On examination, the patient may have proptosis, extending toward the orbital rim. The lower eye- limited extraocular movements, decreased visual lid is then retracted to expose the lateral canthus acuity or vision loss, a sluggish dilated pupil, an tendon, which is cut and released from the orbital afferent pupillary defect, and/or increased intraocu- rim. Despite high IOP, often only a small amount of lar pressure.36,58 Fundoscopy may show optic nerve blood is expressed. The goal pressure should be < 40 pallor or venous dilatation of the disc.57 With this mm Hg.16,59 Complications of a lateral canthotomy disease process, rising IOP compresses the ophthal- include bleeding, infection, and mechanical injury.16 mic and retinal vessels, resulting in retinal ischemia that can occur in as little as 3 to 4 hours.59 Early rec- Globe Rupture And Enucleation ognition of retrobulbar hemorrhage and decompres- sion are the keys to preserving the patient’s vision. Globe rupture may be subtle or may be obvious Bedside US may reveal a hypoechoic lucency deep if the intraocular contents can be seen protruding to the retina. The definitive diagnosis is by CT scan. from a laceration. Globe rupture is a serious injury, 36,59 (See Figure 9, page 14.) a major cause of monocular blindness, and must be Of note, emergency decompression via lateral treated promptly. The patient will often – but not canthotomy may need to precede CT (or arrival of always – complain of pain and decreased visual acu- the ophthalmologist) if there is severe proptosis with ity. Examination may reveal bloody chemosis (swell- diffuse subconjunctival hemorrhage, vision loss, and ing or edema of the conjunctiva), severe subcon- 36 a RAPD. Lateral canthotomy is also indicated once junctival hemorrhage overlying the scleral rupture a definitive diagnosis of retrobulbar hemorrhage has site, a deep or shallow anterior chamber, limitation been made with evidence of increased IOP of extraocular motility, an irregularly shaped pupil, 57 (> 40 mm Hg), acute vision loss, and proptosis. Af- iridodialysis, or exposed uveal tissue that appears ter a simple cleaning of the lids and lateral canthus, brownish-red to brownish-black. the lateral canthus is anesthetized with 1 to 2 mL When globe rupture is present or suspected, of lidocaine (1% or 2%) with epinephrine. A small there should be no further manipulation of the eye. Specifically, any maneuver that may increase IOP,

Figure 7. CT Scan Showing Retinal Detachment Figure 8. Coronal Image Of An Orbital Floor Fracture With Muscle Entrapment

Used with permission from J. Brody, DO. Used with permission from J. Brody, DO.

May 2010 • EBMedicine.net 13 Emergency Medicine Practice © 2010 including tonometry, should not be performed and and polymicrobial in nature.64,65 Treatment options a rigid shield should be placed over the eye for have consisted of early vitrectomy in addition to protection. Globe rupture can be easily visualized on intravitreal, periocular, topical, and systemic antibi- CT. (See Figure 10.) Additional treatment includes otics. In a multicenter, randomized, clinical trial of antiemetics, avoidance of any Valsalva maneuvers 346 eyes treated with either intravitreal gentamicin that can increase IOP, analgesics, tetanus prophy- and clindamycin or balanced saline, the respective laxis as needed, broad-spectrum systemic antibiotics incidences of endophthalmitis were 0.3% and 2.3%.66 to prevent endophthalmitis, and emergent consulta- Other specific antibiotic choices include intravitreal tion with an ophthalmologist. If the patient requires ceftazidime and vancomycin, systemic ceftazidime rapid-sequence intubation for airway management, and vancomycin, periocular ceftazidime and van- succinylcholine may be used after pretreatment with comycin, and topical norfloxacin 0.3%. Intravitreal nondepolarizing and sedative agents.34,60 steroids have also been used, but their use is con- troversial and the supporting data are variable.65 Post-Traumatic Endophthalmitis Suspicion of endophthalmitis necessitates urgent ophthalmologic consult. Endophthalmitis is an inflammation of the in- traocular cavities. Post-traumatic endophthalmitis Penetrating Eye Trauma is a devastating complication of open-globe injury, with a poor visual prognosis. The incidence of Eyelid Lacerations endophthalmitis after an open-globe injury has been There are no randomized, controlled, prospective reported to be between 3.3% and 16.5%. Traditional studies on the treatment of eyelid lacerations. As a risk factors include delayed primary repair and result, ED clinicians rely on the advice of specialty wound closure, a rural setting, the presence of a experts in review articles and textbooks for guidance 61,62 retained IOFB, and disruption of the lens. In a in their treatment. Optimal repair of eyelid lacera- recent, large, consecutive case series conducted over tions is important in maintaining proper function 7 years at 1 institution , the rate of endophthalmitis and health of the eyeball. Intraocular injuries take 63 was 0.4% without IOFB and 3.2% with IOFB. The precedence over injuries to the adnexa (eyelids and most common complaints are pain, photophobia, canalicular system). A thorough examination of the and decreased vision. Pain that is out of proportion entire eye must be done, as described earlier, before to physical findings is a particular red flag for -en repair is undertaken.55 Despite the absence of data, dophthalmitis. Examination may show a variety of experts maintain that eyelid lacerations may be findings, including lid edema, conjunctival erythema repaired up to 48 hours after injury. There seems to and edema, (pus or purulent fluid in the be a low risk of infection and a greater benefit when anterior chamber), vitritis, and retinal periphlebi- repair is achieved under optimal conditions — ie, tis. It is common for these infections to be virulent the most experienced surgeon and the best operating conditions.68,69 The goal of the ED clinician should be to prioritize lid laceration within the spectrum of Figure 9. CT Scan Of Retrobulbar any accompanying ocular or systemic injury. Hemorrhage

Figure 10. Globe Rupture With Lateral Wall Fracture And Intraocular Foreign Body

Used with permission from J. Brody, DO. Used with permission from J. Brody, DO.

Emergency Medicine Practice © 2010 14 EBMedicine.net • May 2010 Partial-thickness lacerations of the eyelids can of the eyelid margin.68 Scar contractures of the skin of be repaired in the same way as other facial lacera- the upper lid may lead to significant , tions. Careful removal of dirt and other particulate which in turn can lead to ocular pain, redness, and matter is necessary to reduce the chance of skin corneal exposure. Patching the eye at night, the use “tattooing.”53 of artificial tears and ointments, and gentle massage Lid margin lacerations are usually quite obvious. may lead to improvement over a 6-month period; if Spasm of the orbicularis muscle may suggest tissue not, scar revision may be necessary.68 loss, but this is extremely rare, and with careful clo- Figure 11 illustrates the process of the 3-step lid sure, the lid will return to normal.53 An ophthalmolo- repair described by Long. The first step is to align the gist should be consulted for lid margin lacerations lid margin by placing a 6-0 silk suture at the “gray because of the significant risk of complications. If, af- line” through both sides of the wound. No knot is ter consulting with an ophthalmologist, it is necessary tied, and an “assistant” pulls on both ends to bring for the ED clinician to repair a lid margin laceration, the eyelid margin into good approximation. The next a technique for closure that involves 3 steps has been step is to repair the tarsus or tarsal plate. The tarsal described by Long (as described below).70 plate of the entire lower lid is 4 mm wide, but the Contrary to most facial suturing (in which su- tarsus width of the upper lid varies from 2 mm to 10 tures in the skin are placed close together and shal- mm. Tarsal closure may be accomplished with 5-0 lowly) lid margin sutures may dehisce if they are not Dexon™ using a D-1 needle and interrupted sutures. placed deeply enough, owing to extensive lid edema Care should be taken not to penetrate the full thick- in the immediate post-repair period.68 Sutures ness of the eyelid to avoid due to protruding placed at the lash line need to be fixed into sutures suture ends. The final step is closure of the anterior away from the edge71 to prevent their dropping lamella (the orbicularis and skin) with deep and wide onto the corneal or conjunctival surface and causing 6-0 silk or nylon sutures. significant irritation or keratitis. General textbooks recommend applying topical antibiotic ointment to Corneal And Scleral Lacerations the laceration following repair, similar to the wide- The slit lamp is the preferred method for examining 53 spread practice for lacerations at other body sites. the cornea and anterior sclera, although a corneal Long et al recommend occlusive patching both to laceration can sometime be seen with the naked eye. 68 support the repair and to reduce reactive edema. (See Figure 12.) Specialized lighting techniques (eg, Complications of inadequate repair include a sclerotic scatter and retroillumination from the fun- notched eyelid, (misdirected eyelashes), dal red reflex) may be necessary to detect very small lagophthalmos (incomplete closure of the eye by the defects.26 The Seidel test used to detect full-thickness eyelid), and corneal exposure. A notched eyelid is laceration was described earlier under Physical largely a cosmetic issue and may occur because of Examination (see page 4). The care of corneal and wound dehiscence or poor repair. It may resolve over scleral lacerations requires the technical skill of an 6 months, but if not, elective repair using block exci- ophthalmologist. sion and meticulous closure by an oculoplastic sur- geon is usually curative. Trichiasis may occur because Canalicular Injuries hair follicles were inadvertently incorporated into the In contrast to eyelid lacerations, which are usually wound during repair or because of microanatomic obvious, canalicular injuries can easily be missed. changes within the lid margin. Again, this can be re- paired by block excision or cryotherapy and rotation Figure 12. Corneal Laceration

Figure 11. Long’s 3-Step Closure Of Eyelid Laceration

Direct Closure of a Marginal Eyelid Laceration Placement of initial Partial-thickness Margin sutures tied margin suture lamellar sutures through skin sutures in the tarsus

tarsus tarsal skin sutures skin orbicularis muscle sutures eyelid retractors Used with permission from The University of Iowa, EyeRounds.org.

May 2010 • EBMedicine.net 15 Emergency Medicine Practice © 2010 Clinical Pathway For Blunt Eye Trauma

History (how, when, where, No further examination and what) Place eyeshield Call for ophthalmology consult Obvious globe rupture? Consider CT scan STOP! YES Send to OR (Class I)

NO

Perform lateral canthotomy Bedside ultrasound / CT Call for ophthalmology Proptosis? YES Retrobulbar hematoma scan consult Send to OR (Class I) NO

Entrapment of ocular Primary or delayed repair muscles? YES CT scan Orbital fracture (Class II)

NO

Traumatic miosis Pupillary abnormality? YES Traumatic mydriasis Refer to ophthalmologist Anterior uveitis (See Table 2, page 9) Iridodialysis NO

Rapid afferent pupillary Optic nerve injury YES Urgent ophthalmologic defect? Large retinal tear consult Large vitreous injury

NO Provide supportive care (Class II) Anterior chamber injury? YES Hyphema Administer topical steroids Observe for rebleeding Consult ophthalmology NO (See Table 2, page 9)

Traumatic cataract Dislocated lens Urgent ophthalmologic Abnormal vision? YES Vitreous hemorrhage consult Retinal detachment NO

High Acute glaucoma Intraocular pressure? Emergent ophthalmologic Hemorrhage within globe Low consult for medical treat- ment and/or possible surgery (Class I) Ruptured globe (See Table 2, page 9)

Fundoscopy

See next page for Class of Evidence definitions.

Emergency Medicine Practice © 2010 16 EBMedicine.net • May 2010 Clinical Pathway For Penetrating Eye Trauma

History (how, when, where, and what)

Impaled object? YES STOP! Stabilize object Call for ophthalmology consult NO Remove object in OR (Class II)

Full eye examination

Call for ophthalmology Evaluate for canalicular consult Lid laceration? YES injury Repair within 24 to 48 hours (Class II) NO

Call for ophthalmology consult Corneal laceration? YES Seidel test Perform emergent repair (Class II)

NO

Intraocular foreign body? Metallic? YES CT scan

NO

Ophthalmology consult To OR for removal and Vegetal? MRI repair Class II

Class Of Evidence Definitions

Each action in the clinical pathways section of Emergency Medicine Practice receives a score based on the following definitions.

Class I Class II Class III Indeterminate tatives from the resuscitation • Always acceptable, safe • Safe, acceptable • May be acceptable • Continuing area of research councils of ILCOR: How to De- • Definitely useful • Probably useful • Possibly useful • No recommendations until velop Evidence-Based Guidelines • Proven in both efficacy and • Considered optional or alterna- further research for Emergency Cardiac Care: effectiveness Level of Evidence: tive treatments Quality of Evidence and Classes • Generally higher levels of Level of Evidence: of Recommendations; also: Level of Evidence: evidence Level of Evidence: • Evidence not available Anonymous. Guidelines for car- • One or more large prospective • Non-randomized or retrospec- • Generally lower or intermediate • Higher studies in progress diopulmonary resuscitation and studies are present (with rare tive studies: historic, cohort, or levels of evidence • Results inconsistent, contradic- emergency cardiac care. Emer- exceptions) case control studies • Case series, animal studies, tory gency Cardiac Care Committee • High-quality meta-analyses • Less robust RCTs consensus panels • Results not compelling and Subcommittees, American • Study results consistently posi- • Results consistently positive • Occasionally positive results Heart Association. Part IX. Ensur- tive and compelling Significantly modified from: The Emergency Cardiovascular Care ing effectiveness of community- Committees of the American wide emergency cardiac care. Heart Association and represen- JAMA. 1992;268(16):2289-2295. This clinical pathway is intended to supplement, rather than substitute for, professional judgment and may be changed depending upon a patient’s individual needs. Failure to comply with this pathway does not represent a breach of the standard of care. Copyright © 2010 EB Practice, LLC d.b.a. EB Medicine. 1-800-249-5770. No part of this publication may be reproduced in any format without written consent of EB Practice, LLC d.b.a. EB Medicine.

May 2010 • EBMedicine.net 17 Emergency Medicine Practice © 2010 Injuries may be due to laceration, avulsion, or shear- tant to assume an IOFB until proven otherwise. ing.72 A small, retrospective case series from a single Suspicious physical findings include hemorrhage institution revealed that 66% of dog bite injuries to over the sclera, localized corneal edema, and a the face involved the canalicular system, as com- nonsurgical hole in the iris. Other determinants pared with 34% in the group not bitten by a dog.73 of injury in IOFBs are as follows: the shorter the A high index of suspicion needs to be maintained entry wound, the higher the risk of retinal injury; for any laceration/avulsion in the area of the medial objects entering through the sclera are more likely canthus. Evaluation of the canalicular structures is to cause damage than those entering through the performed by the ophthalmologist and requires an cornea; and the sharper the foreign body, the less awake, cooperative patient. Repair may be carried out destruction it causes. More than 50% of IOFBs end 24 to 48 hours after the initial injury.68 The standard up in the retina or choroid. The composition of the treatment is to use silicone bicanalicular intubation, IOFB is also important, not only because of the with the tubing left in place for 2 months.68 Complica- type of imaging required, but also because for- tions include dislodgment of the tubing, but the major eign bodies that contain iron or copper may cause complication is failure to diagnose canalicular system severe chemical reactions that will lead to inflam- laceration at the time of the injury.68,72 mation and further damage. Data from before 1990 cited in Mester and Kuhn lists missed IOFBs Superficial Foreign Body as representing 56% of all ocular trauma-related Foreign bodies can become lodged under the lids malpractice claims.7 or in the superficial layers of the cornea. Super- Newer helical CT scanners are able to detect ficial corneal foreign bodies can often be washed IOFBs as small as 0.048 mm. Ultrasound is an excel- away with saline irrigation, and those under the lent modality in experienced hands but may miss lids may be removed with cotton-tipped appli- small, wooden, or organic matter. MRI is very sensi- cators. Foreign bodies embedded in the cornea tive but cannot be used if the IOFB contains metal, should be removed with a spud device or a 25- or since the magnet may cause shifting of the fragment 27-gauge needle, preferably under magnification. and additional intraocular damage.7 Rust rings from iron-containing foreign bodies Management of an IOFB involves referral to a may be removed at the initial visit by the ED clini- surgeon for what is likely to involve comprehen- cian or later at follow-up. Aftercare is similar to sive globe reconstruction, including removal of the that for corneal abrasions.16 foreign body, wound closure, and treatment of each individual tissue lesion. Vision does not improve Intraocular Foreign Bodies simply by removing the IOFB, and inappropriate or Of the patients with an IOFB, 20% have no pain on incomplete surgical intervention can cause iatro- 7 presentation and may have normal or near-normal genic injury. Endophthalmitis has been reported to vision. If the history is suggestive, it is impor- be present in 90% to 100% of injuries with IOFB at

Risk Management Pearls For Traumatic Ocular Emergencies

1. Do no harm: do not cause iatrogenic injury by 6. Do not contaminate a penetrating injury by failing to protect an open-globe injury with an administering nonsterile eyedrops. eyeshield. If you suspect an open-globe injury, stop the physical examination immediately and 7. Avoid use of eye patches in corneal abrasions protect the eye with an eyeshield. less than 10 mm.

2. Always check IOP unless you suspect an open- 8. Always include an assessment of optic nerve globe injury. function by using the swinging flashlight test (see Figure 4, page 6) when treating a patient 3. The management of IOFBs represented over 56% with an ophthalmologic complaint. of ocular-related malpractice claims. 9. Use topical NSAIDs in order to minimize the 4. Get the most experienced person for the eyelid need for systemic pain medications and their or canalicular lacerations, and remember that associated complications. the repair can be delayed for 24 to 48 hours. 10. Consider admission for patients with a hyphema 5. Dog bites to the eye result in a high incidence of > 50%, since these patients are at significantly canalicular injuries. greater risk for complications.

Emergency Medicine Practice © 2010 18 EBMedicine.net • May 2010 initial evaluation; however, in an early study, Mieler Regarding the use of prophylactic antibiot- et al showed that positive vitreous cultures did not ics in globe rupture, there are no randomized, translate into clinical endophthalmitis in 7 out of 19 prospective studies of open-globe injuries treated cases.74 High-risk lesions are IOFBs that consist of with or without systemic antibiotics, nor are there organic material or copper and wounds with soil likely to be any such studies in the future. For contamination.7 ED clinicians, the practice consensus is to begin Early surgery has been considered the standard prophylactic IV antibiotics, with choices including of care for IOFBs for a number of years, since it is ceftazidime in combination with vancomycin or a believed to decrease infection and improve visual fluoroquinolone. outcome. Historically, post-injury infection rates have been 4% to 8%. Andreoli et al reported that sur- Summary gery within 24 hours, with removal of the IOFB plus systemic antibiotic therapy, resulted in an infection The evaluation of blunt and penetrating injuries to 63 rate of 3.2%. A recent retrospective case series from the orbit represents a challenge to the practicing the military showed that early closure of the globe ED clinician. The history and physical examina- was more important than removal of the IOFB in tion need to be as complete as possible while re- terms of visual outcome. These patients underwent maining vigilant for open-globe injuries and IOFB. early surgery for wound closure, but IOFB removal Some injuries are relatively common and self-lim- was delayed as long as 4 weeks, with prophylactic iting, such as simple corneal abrasions or trau- 75 administration of topical and systemic antibiotics. matic uveitis. Other less-common injuries such Besides vision loss and infection, other complica- as hyphema and traumatic cataracts also usually tions following surgery include epimacular prolifer- have a good outcome. However, other types of ation and retinal detachment, which can negatively ocular trauma such as open-globe injuries, certain affect vision. With IOFB, certain factors such as BB or metallic IOFBs, and traumatic glaucoma can be shotgun pellets are associated with a poor progno- devastating and threaten the patient’s vision. The sis; however, visual acuity of 20/40 or better can be ED clinician should know when to safely initiate 7 expected in 71% of cases. appropriate treatment as well as when to stop the examination abruptly, stabilize the eye, and call in Sympathetic Ophthalmia an ophthalmologist. Sympathetic ophthalmia is a rare granulomatous uveitis that occurs in the uninvolved eye after injury Case Conclusions or surgery to the other eye. The incidence has been 60 estimated to be 0.03 per 100,000 patients. Onset is The small “gray dot” in the sclera and “jiggling” eyeball usually 3 months after the injury or surgery but has suggested globe rupture despite the young man’s near- been reported to occur years after the original insult. normal visual acuity and the absence of pain. The physical It is thought to be an autoimmune process and may examination was stopped immediately, and a plastic present acutely or insidiously. The diagnosis is made shield was placed over the eye to protect it from possible based on a history of previous injury or surgery plus iatrogenic injury. The patient was sent for CT scan, which findings of uveitis. Treatment consists of high-dose revealed a BB pellet located next to the right optic nerve systemic steroids or other immunosuppressive 77 on the medial side of the retrobulbar space. Ophthalmol- therapy. ogy was contacted, and the patient was taken to the OR, where the BB was successfully removed. At 6-month Controversies/Cutting Edge follow-up, his vision was normal. As for the second patient, examination of the left eye Rebleeding is the dreaded complication of acute revealed a visual acuity of 20/30. The conjunctiva was hyphema. Antifibrinolytic agents have been used injected, and the pupil was 2 mm in diameter and round. in children with variable success, but their use in Anesthetic drops relieved the pain to some degree, but she adults is controversial. Some studies have report- still had pain when a light was shone in her other eye. ed a decrease in the rebleeding rates when Fluorescein staining revealed a 3-mm corneal abrasion at aminocaproic acid and tranexamic acid are used, 4 o’clock. No foreign bodies were noted on lid eversion. but these same agents have serious potential Results of slit-lamp and IOP examinations were normal. side effects, including nausea, vomiting, muscle The patient did not wear contact lenses. A cyclople- cramps, conjunctival suffocation, headache, gic agent was instilled in the eye, and the patient was rash, pruritus, dyspnea, toxic confusional states, discharged on topical antibiotic drops as well as NSAID arrhythmias, and systemic hypotension. These drops, along with back-up oral analgesics. The following agents should be used only after consultation day the patient followed up with her ophthalmologist, at with an ophthalmologist.41 which time a healing superficial corneal abrasion with some mild anterior uveitis was found.

May 2010 • EBMedicine.net 19 Emergency Medicine Practice © 2010 References 16. Knoop KJ, Dennis WR, Hedges JR. Ophthalmologic proce- dures. In: Roberts JR and Hedges JR, eds. Clinical Procedures in Emergency Medicine. 4th ed. Philadelphia, Pa: WB Saun- Evidence-based medicine requires a critical ap- ders; 2004:1241-1279. (Book chapter) praisal of the literature based upon study methodol- 17.* Guluma K. An evidence-based approach to abnormal vision. ogy and number of subjects. Not all references are Emergency Medicine Practice. 2007;9(9). (Evidence-based equally robust. The findings of a large, prospective, review article) 18. Blaivas M, Theodoro D, Sierzenski PR. A study of bedside randomized, and blinded trial should carry more ocular ultrasonography in the emergency department. Acad weight than a case report. Emerg Med. 2002;9(8):791-799. (Small, prospective, observa- To help the reader judge the strength of each tional study) reference, pertinent information about the study, 19. Blaivas M. Bedside emergency department ultrasonogra- such as the type of study and the number of patients phy in the evaluation of ocular pathology. Acad Emerg Med. 2000;7(8):947-950. (Report of 2 cases) in the study, will be included in bold type following 20. Joseph DP, Pieramici DJ, Beauchamp, NJ. Computed tomog- the reference, where available. In addition, the most raphy in the diagnosis and prognosis of open-globe injuries. informative references cited in this paper, as deter- Ophthalmology. 2000;107(10):1899-1906. (Review article) mined by the authors, are noted by an asterisk (*) 21. Bilaniuk L, et al. Magnetic resonance imaging of the orbit. Radiol Clin N Am. 1987;25(3):509-528. (Review article) 1. Nawar EW, Niska RW, Xu J. National hospital ambulatory 22. Kuhn F, Masiak R, Mann L, et al. The ocular trauma score Review of medical care survey: 2005 emergency department summary. (OTS). Ophthalmol Clin N Am. 2002;15:163-165. ( eye injury scoring system National Health Statistics Reports. No 6, 2007. National Center ) for Health Statistics, Hyattsville, MD. (Epidemiologic sur- 23. Unver YB, Kapran Z, Acar N, et al. Ocular trauma score in Retro- vey) open-globe injuries. J Trauma. 2009;66(4):1030-1032. ( spective case series 2. Pitts SR, Niska RW, Xu J, et al. National hospital ambulatory ) medical care survey: 2006 emergency department summary. 24. Schmidt GW, Broman AT, Hindman HB, et al. Vision sur- National Health Statistics Reports. No 7, 2008. National Center vival after open globe injury predicted by classification and for Health Statistics, Hyattsville, MD. (Epidemiologic sur- regression tree analysis. Ophthalmology. 2008;115(1):202-209. Retrospective cohort study vey) ( ) 3. Kuhn F, Morris R, Mester V, et al. Epidemiology and socio- 25. Man CYW, Steel D. Visual outcome after open globe injury: a economics. Ophthalmol Clin N Am. 2002;15:145-151. (Review comparison of two prognostic models – ocular trauma score article) and the classification and regression tree. Eye. 2010;24:84-89. Retrospective cohort study 4. Harris PM. Nonfatal Occupational Injuries Involving the Eyes, Published online Feb 20, 2009. ( ) 2004. Bureau of Labor Statistics; published online August 30, 26. Hamill MB. Corneal and scleral trauma. Ophthalmol Clin N 2006 (www.bls.gov). (Epidemiologic survey) Am. 2002;15:185-194. 5. Forrest KY, Cali JM. Epidemiology of lifetime work-related 27. Wilson SA, Last A. Management of corneal abrasions. Am Review article eye injuries in the U.S. population associated with one or Fam Phys. 2004;70(1):123-128. ( ) more lost days of work. Ophthalmic Epidemiol. 2009;16(3):156- 28. Aslam SA, Sheth, HG, Vaughn AJ. Emergency management Retrospective 162. (Epidemiologic prevalence study) of corneal injuries. Injury. 2006;28:594-597. ( case study 6. Xiang H, Stallones L, Chen G, et al. Work-related eye injuries ) treated in hospital emergency departments in the US. Am J 29. Dart JK. Predisposing factors in microbial keratitis: the sig- Ind Med. 2005;48(1):57-62. (Epidemiologic survey) nificance of contact lens wear.Br J Ophthalmol. 1988;72:926- Consecutive series review 7.* Mester V, Kuhn F. Intraocular foreign bodies. Ophthalmol Clin 930 ( ) N Am. 2002;15:235-242. (Review article) 30. Fleiszig SM, Efron N, Pier GB. Extended contact lens wear 8. Conn JM, Annest JL, Gilchrist J, Ryan G. Injuries from paint- enhances Pseudomonas aeruginosa adherence to human cor- ball game related activities in the United States, 1997-2001. neal epithelium. Invest Ophthalmol Visual Sci. 1992;33:2908- Inj Prev. 2004;10(3):139-143. (Retrospective case series) 2916. 9. Sinclair SA, Smith GA, Xiang H. Eyeglasses-related injuries 31.* Turner A, Rabiu M. Patching for corneal abrasion. Cochrane Evidence-based treated in US emergency departments in 2002–2003. Ophthal- Database Syst Rev. 2006;2:CD004764 (pub2). ( review mic Epidemiol. 2006;13(1):23-30. (Epidemiologic survey) ) 10. Graham JD, Thompson KM, Goldie SJ, et al. The cost 32. Verma A, Ehrenhaus MP. Corneal erosion, recurrent. Emedi- Review article effectiveness of airbags by seating position. JAMA. cine. www.webmd.com. Retrieved 4/25/08. ( ) 1997;278(17):1419-1425. (Population-based convenience 33. Watson SL, Barker NH. Interventions for recurrent cor- sample) neal erosions. Cochrane Database Syst Rev. 2007;4:CD001861 Evidence-based review 11. American Academy of Ophthalmology Joint Policy Statement (pub2). ( ) on Protective Eye Wear, 2003. Am Acad Ophthalmol. San Fran- 34. Dalma-Weiszhausz JD, Dalma A. The uvea in ocular trauma. Review article cisco, CA. (Policy statement by professional organization) Ophthalmol Clin N Am. 2002;15:205-213. ( ) 12. Kuhn F, Morris R, Witherspoon CD. Birmingham eye trauma 35.* Dargin JM, Lowenstein RA. The painful eye. Emerg Clin N Review article terminology (BETT): terminology and classification of me- Am. 2008;26(1):199-216. ( ) chanical eye injuries. Ophthalmol Clin N Am. 2002;15:139-143. 36.* Bord SP, Linden J. Trauma to the globe and orbit. Emerg Med Review article (Review of new terminology for eye trauma) Clin N Am. 2008;26:97-123. ( ) 13. Limmer D, O’Keefe MF, Dickinson ET. Brady Emergency Care. 37. Wightman JM, Hamilton GC. Red and painful eye. In: Marx 10th ed. Upper Saddle River, NJ: Pearson/Prentice Hall; JA, et al, eds. Rosen’s Emergency Medicine: Concepts and Clini- 2005. (Textbook) cal Practice. 6th ed. Philadelphia Pa: Mosby Inc.; 2006:283-298 Book chapter 14.* Harlan JB, Pieramici DJ. Evaluation of patients with ocular ( ) trauma. Ophthalmol Clin N Am. 2002;15:153-161. (Review 38. DeLeon-Ortega JE, Girkin CA. Ocular trauma-related glau- Review article article) coma. Ophthalmol Clin N Am. 2002;15:215-223. ( ) 15. Juang PSC, Rosen P. Ocular examination techniques for the 39. Martini E, et al. Ocular blood flow evaluation in injured and Small emergency department. J Emerg Med. 1997;15(6):793-810. healthy fellow eyes. Eur J Ophthal. 2005;15(1):48-55. ( case series (Review article) )

Emergency Medicine Practice © 2010 20 EBMedicine.net • May 2010 40. Girkin CA, et al.Glaucoma following penetrating ocular 62. Lieb DF, Scott IU, Flynn HW Jr, Miller D, Feuer WJ. Open trauma: a cohort study of the United States Eye Injury Reg- globe injuries with positive intraocular cultures: factors istry. Am J Ophthal. 2005;139(1):100-105. (Prospective cohort influencing final visual acuity outcomes.Ophthalmology. study) 2003;110(8):1560-1566. (Retrospective case series) 41.* Vass C, Hirn C, Sycha T, et al. Medical interventions for 63. Andreoli CM, Andreoli MT, Klock CE, et al. Low rate of primary open angle glaucoma and . endophthalmitis in a large case series of open globe injuries. Cochrane Database Syst Rev. 2007;4: CD003167 (pub3). Ophthalmology. 2009;147:601-608. (Retrospective case series) (Evidence-based review) 64. Ramakrishnan R, Bharathi MJ, Shivkumar C, et al. Micro- 42. Salmon JF. Glaucoma. Riordan-Eva P, Whitcher JP. Glau- biological profile of culture-proven cases of exogenous and coma. In: Riordan-Eva P, Whitcher JP, eds. Glaucoma. Vaughn endogenous endophthalmitis: a 10-year retrospective study. & Asbury’s General Ophthalmology. 17th ed. New York, NY: Eye. 2009;23(4):945-956. (Retrospective case study) McGraw-Hill; 2008:212-229. (Book chapter) 65. Danis RP. Endophthalmitis. Ophthalmol Clin N Am. 43.* Savage HI, Gharaibeh A, Mathew M, Scherer RW. Medical 2002;15:243-248. (Review article) interventions for traumatic hyphema (protocol). Cochrane 66. Soheilian M, Rafati N, Mohebbi MR, et al. Prophylaxis of Database of Syst Rev. 2005;(3):CD005431. (Evidence-based acute posttraumatic bacterial endophthalmitis: a multicenter, review) randomized clinical trial of intraocular antibiotic injection 44. Kuhn F, Mester V. Anterior chamber abnormalities and cata- (report 2). Arch Ophthalmol. 2007;125:460-465. (Prospective, ract. Ophthalmol Clin N Am. 2002;15:195-203. (Review article) randomized clinical trial) 45. Tingey DP, Shingleton BJ. Glaucoma associated with ocular 67. Liang EC, Lin M. Tricks of the trade for lacerations. Critical trauma. In: Yanoff M, Duker JS, eds. Yanoff & Duker Ophthal- Decisions In Emergency Medicine. 2009;23:8.1-19. (Review mology. 3rd ed. St. Louis, Mo: Mosby Inc; 2008:1095-1293. article) (Book chapter) 68. Long J, Tann T. Adnexal trauma. Ophthalmol Clin N Am. 46. Albiani DA, et al. Tranexamic acid in the treatment of pedi- 2002;15:179-184. (Review article) atric traumatic hyphema. Can J Ophthalmol. 2008;43:428-431. 69. Brown DJ, Jaffe JE, Henson JK. Advanced laceration manage- (Retrospective cohort study) ment. Emerg Med Clin N Am. 2007;25:83-99. (Review article) 47. Seth RK, et al. Cataract secondary to electrical shock from 70. Eyelid trauma and reconstruction techniques. In: Yanoff Taser® gun. J Cataract Refract Surg. 2007;33:1664-1665. (Case M, Duker JS, eds. Yanoff & Duker Ophthalmology. 3rd ed. St. report) Louis, Mo: Mosby Inc; 2008:1393-1443. (Book chapter) 48. Das S, et al. Traumatic fracture of posterior chamber 71. Ali SN, Budny PG. “Minding the ends”: a simple tech- intraocular lens. J Cataract Refract Surg. 2007; 33:2151-2152. nique for repair of lower eyelid lacerations. Emerg Med J. (Case report) 2004;21(2):263 (Case report) 49. Yang CS, et al. Air bag associated posterior segment ocular 72. Della Rocca DA, Ahmad SM, Della Rocca RC. Direct repair trauma. J Chin Med Assoc. 67(8):425-431. (Case report) of canalicular lacerations. Facial Plast Surg. 2007;23(3):149- 50. Kenney KS, Fanciullo LM, Automobile air bags: friend or 155. (Review of repair techniques) foe? A case of air bag-associated ocular trauma and a related 73. Savar A, Kirszrot J, Rubin PAD. Canalicular involvement literature review. Optometry. 2005;76:382-386. (Case report) in dog bite related eyelid lacerations. Ophthal Plast Reconstr 51. Pahk, PJ, Adelman RA. Ocular trauma resulting from paint- Surg. 2008;24(4):296-298. (Retrospective case series) ball injury. Graefes Arch Clin Exp Ophthalmol. 2009;247(4):469- 74. Mieler WF, Ellis MK, Williams DF, et al. Retained intraocu- 475. (Retrospective chart review) lar foreign bodies and endophthalmitis. Ophthalmology. 52. Pieramici, DJ. Vitreoretinal trauma. Ophthalmol Clin N Am. 1990:97:1532-1538. (Retrospective case series) 2002;15:225-234. (Review article) 75. Colyer MH, Weber ED, Weichel ED, et al. Delayed intraocu- 53. Augsburger J, Asbury T. Ocular & orbital trauma. In: lar foreign body removal without endophthalmitis during Riordan-Eva P, Whitcher JP, eds. Vaughan & Asbury’s General Operation Iraqi Freedom and Operation Enduring Freedom. Ophthalmology, Seventeenth Edition. New York, NY: McGraw- Ophthalmology. 2007;114:1439-1447. (Retrospective interven- Hill Companies. 2008:368-376. (Book chapter) tional case series) 54.* Harris GJ. Orbital blow-out fractures: surgical timing and 76. Kilmartin DJ, Dick AD, Forrester JV. Prospective surveillance technique. Eye. 2006;20:1207-1212. (Evidence-based review) of sympathetic ophthalmia in the UK and Republic of Ire- 55. Long J, Tann T. Orbital trauma. Ophthalmol Clin N Am. land. Curr Opinion Ophthalmol. 2000;11(5):372-378. (Prospec- 2002;15:249-253. (Review article) tive survey) 56. Brady SM, Mcmann M, Mazzoli O, et al. The diagnosis and 77. Damico FM, Kiss S, Young LH. Sympathetic ophthalmia. management of orbital blowout fractures: update 2001. Am J Semin Ophthalmol. 2005;20(5):191-197 (Review article) Emerg Med. 2001;19(2):147-154. (Review article) 57. Cruz AAV, Eichenberger GCD. Epidemiology and man- agement of orbital fractures. Curr Opinion Ophthalmol. Practice Recommendations 2004;15:416-421. (Review article) 58.* Ceallaigh PO, Ekanaykaee K, Beirne CJ, Patton DW. Diagno- sis and management of common maxillofacial injuries in the Available Online! emergency department. Part 4: orbital and midface fractures. Emerg Med J. 2007;24:292-293. (Brief review article) You asked and we listened! The one-page Evidence- 59.* Babineau MR, Sanchez LD. Ophthalmologic procedures Based Practice Recommendations sheet that accom- in the emergency department. Emerg Med Clin N Am. 2008;26:17-34. (Review article) panies your monthly issue is now available online 60. Brunette DD. Ophthalmology. In: Marx JA, Hockberger R, instead of in print. To download your copy, visit Walls R, eds. Rosen’s Emergency Medicine: Concepts and Clinical http://www.ebmedicine.net/topics, click the title Practice. 6th ed. Philadelphia, Pa: Mosby Inc; 2006:1044-1065. of the article, and click “Practice Recommendations (Book chapter) (key points from the issue)” in the Table of Contents 61. Chaudhry IA, Shamsi FA, Al-Harthi E, et al. Incidence and visual outcome of endophthalmitis associated with intraocu- below the Abstract. lar foreign bodies. Arch Clin Exp Ophthalmol. 2008;246:181- 186. (Retrospective case review)

May 2010 • EBMedicine.net 21 Emergency Medicine Practice © 2010 CME Questions 7. Occlusive patching for corneal abrasion should be applied to: a. Abrasions greater than 10 mm T a ke This Test Online! b. All corneal abrasions c. Corneal abrasions associated with an IOFB Current subscribers receive CME credit absolutely d. Infected corneal abrasions free by completing the following test. Monthly on- line testing is now available for current and archived 8. The Seidel test: issues. Visit http://www.ebmedicine.net/CMET a ke This Test Online! a. Assesses visual fields today to receive your free CME credits. Each issue TM b. Assesses visual acuity includes 4 AMA PRA Category 1 Credits , 4 ACEP c. Tests for ocular motility Category 1 credits, 4 AAFP Prescribed credits, and 4 d. Uses fluorescein dye to detect an aqueous AOA Category 2B credits. fluid leak

1. The ophthalmologic history should include: 9. All of the following may be part of the stan- a. Location of injury dard acceptable treatment for corneal abrasions b. Mechanism of injury EXCEPT: c. Prior ocular surgery a. Eye patch d. Time of injury b. Oral narcotics e. All of the above c. Topical antibiotics d. Topical NSAIDs 2. The swinging flashlight test: a. Detects blood in the anterior chamber 10. After trauma, the presence of cells and flare in b. Is a test of intraocular pressure the anterior chamber on slit-lamp examination c. Is a test of visual acuity together with pain is diagnostic of: d. May detect abnormalities of the optic nerve a. Traumatic iridocyclitis b. Traumatic glaucoma 3. A screwdriver is protruding from your pa- c. Hyphema tient’s eye. You should: d. Traumatic retinal detachment a. Call ophthalmology and then remove it b. Give antibiotics and then remove it 11. The earliest symptom of hyphema rebleeding c. Remove it immediately is: d. Stabilize the screwdriver to prevent a. Eye pain movement and call ophthalmology b. Decreased vision c. Nausea and vomiting 4. Ultrasound is useful for detecting all EXCEPT: d. Periorbital swelling a. A dislocated lens b. Organic-matter IOFBs 12. Emergent ophthalmology consultation is re- c. Retinal detachment quired for all of the following EXCEPT: d. Retrobulbar hemorrhage a. Globe rupture e. Vitreous hemorrhage b. Post-traumatic endophthalmitis c. Traumatic glaucoma 5. Risk factors for the development of en- d. Uveitis dophthalmitis include all EXCEPT: a. Delayed repair of a penetrating injury 13. In the setting of retrobulbar hematoma, in- b. A disrupted lens creased IOP can cause permanent ischemic c. Injury in a rural setting changes in as little as: d. A retained IOFB a. Less than 5 minutes e. Retrobulbar hemorrhage b. 10 to 15 minutes c. 15 to 30 minutes 6. Factors that affect visual outcome after injury d. 3 to 4 hours include all EXCEPT: a. An associated orbital fracture 14. The first step in the treating globe rupture is: b. Vision at time of injury a. Assess IOP c. Presence of globe rupture b. Examine the eye carefully d. Presence of RAPD c. Give tetanus prophylaxis e. Retinal detachment d. Place eyeshield over the orbit

Emergency Medicine Practice © 2010 22 EBMedicine.net • May 2010 EM Practice Guidelines Update: Stay Up-To-Date In Less Than An Hour Per Month

In October 2009, we launched a new free benefit for and include an editorial commentary, if indicated. Emergency Medicine Practice subscribers: EM Practice We hope this free, online, interactive publication will Guidelines Update. This monthly online publica- become a valued resource to our Emergency Medicine tion was created to help emergency clinicians stay Practice subscribers and will help facilitate best prac- current with clinical policies and practice guide- tice in emergency care. To begin staying up-to-date lines that can assist clinical decision-making in the in less than an hour per month, download the cur- emergency department. The practice guidelines rent issue, “Current Guidelines For Diagnosis And are drawn from a variety of specialty sources, are Management Of Bronchiolitis In The Emergency analytically reviewed using a systematic approach, Department,” at www.ebmedicine.net/EMPGU.

Emergency Medicine Practice Special Report Review, by Joseph Sabato, Jr., MD, FACEP

“Hospital Planning For Terrorist Disasters: Education and ongoing training are emphasized A Community-Wide Program” as key areas for successfully implementing plans. Security, the development of alternative care site by Solisis Deynes, MD, MPH; Christopher Kahn, MD, strategies, psychological support services, surveil- MPH; and Kristi L. Koenig, MD, FACEP, FIFEM lance, media, volunteers, personal protective equip- ment, decontamination, communications, and surge Terrorism and healthcare are inescapably linked. A capacity are discussed in this report as important terrorist event would lead to casualties that hospi- components of the plan and response. tals would need to care for. Hospitals themselves One of the major problems with disaster plan- may be seen as high-value and high-impact targets. ning is that it is a lot like insurance – hard to pay Although the likelihood of any given hospital being attention to until you need it. Preparing for terrorist impacted by a terrorist event is low, intelligence events is a galvanizing opportunity that can bring leaders have recently predicted an event as “inevita- the concern, energy, and resources of a variety of ble.” Successful response to terrorism would ideally community parties together. Planning for terror- result from community-wide planning. “Hospital ist events provides a tremendous opportunity to Planning for Terrorist Disasters: A Community-Wide strengthen overall disaster preparedness and use Program” provides a foundation for planning based the resultant attention to develop improved com- on accepted key principles and the experience of munity response. Learning from the more common prior events. non-terrorist events and the application to planning As the report states, disasters are defined by and response is the key. The special report, “Hospi- whether the health and medical needs of the victims tal Planning For Terrorist Disasters: A Community- are able to be met. The phases of comprehensive Wide Program,” presents an excellent foundation for emergency management, mitigation (prevention), hospital planning for terrorism that can be readily preparedness (planning), response (immediate adapted to a number of threats. reaction to event), and recovery are reviewed. The principles of incident command and the Hospital In- Reviewed by cident Command System (HICS) are introduced and the need for multi-agency coordination in terrorist events is emphasized. Disaster triage is covered extensively with an emphasis on the use of the START triage. A key con- cept is that triage is a dynamic and not a static event. Anticipating and planning for the waves of casualties Joseph Sabato, Jr., MD, FACEP that occur in disasters, with the understanding that Assistant Professor the most serious casualties often arrive after the least Department of Emergency Medicine injured, is of critical importance. In addition, in the Director of Field Operations days beyond the initial event, planning for the vol- University of Florida College of Medicine Jacksonville ume of patients with exacerbation of chronic condi- Jacksonville, Florida tions in a disrupted healthcare system is important.

For more information on “Hospital Planning For Terrorist Disasters: A Community-Wide Program,” please visit www.ebmedicine.net/disasters

May 2010 • EBMedicine.net 23 Emergency Medicine Practice © 2010 Physician CME Information Date of Original Release: May 1, 2010. Date of most recent review: December 1, 2009. Termination date: May 1, 2013. Accreditation: EB Medicine is accredited by the ACCME to provide continuing medical education for physicians. Credit Designation: EB Medicine designates this educational activity for a maximum of 48 AMA PRA Category 1 CreditsTM per year. Physicians should only claim credit commensurate with the extent of their participation in the activity. ACEP Accreditation: Emergency Medicine Practice is approved by the American College of Emergency Physicians for 48 hours of ACEP Category 1 credit per annual subscription. AAFP Accreditation: Emergency Medicine Practice has been reviewed and is acceptable for up to 48 Prescribed credits per year by the American Academy of Family Physicians. AAFP Accreditation begins August 1, 2009. Term of approval is for 1 year from this date. Each issue is approved for 4 Prescribed credits. Credits may be claimed for 1 year from the date of this issue. AOA Accreditation: Emergency Medicine Practice has been approved for 48 Category 2B credit hours per year by the American Osteopathic Association. Needs Assessment: The need for this educational activity was determined by a survey of medical staff, including the editorial board of this publication; review of morbidity and mortality data from the CDC, AHA, NCHS, and ACEP; and evaluation of prior activities for emergency physicians. Target Audience: This enduring material is designed for emergency medicine physicians, physician assistants, nurse practitioners, and residents. Goals & Objectives: Upon completion of this article, you should be able to: (1) demonstrate medical decision-making based on the strongest clinical evidence; (2) cost-effectively diagnose and treat the most critical ED presentations; and (3) describe the most common medicolegal pitfalls for each topic covered. Discussion of Investigational Information: As part of the newsletter, faculty may be presenting investigational information about pharmaceutical products that is outside Food and Drug Administration-approved labeling. Information presented as part of Group subscriptions this activity is intended solely as continuing medical education and is not intended to promote off-label use of any pharmaceutical product. are available offering Faculty Disclosure: It is the policy of EB Medicine to ensure objectivity, balance, independence, transparency, and scientific rigor in all CME-sponsored educational activities. All faculty participating in the planning or implementation of a sponsored substantial discounts activity are expected to disclose to the audience any relevant financial relationships and to assist in resolving any conflict of interest that may arise from the relationship. Presenters must also make a meaningful disclosure to the audience of their off the regular price. discussions of unlabeled or unapproved drugs or devices. In compliance with all ACCME Essentials, Standards, and Guidelines, all faculty for this CME activity were asked to complete a full disclosure statement. The information received is as follows: Dr. Alteveer, Dr. Lahmann, Dr. Sharma, Dr. Walther, and their Please contact related parties report no significant financial interest or other relationship with the manufacturer(s) of any commercial product(s) discussed in this educational Robert Williford, presentation. Method of Participation: Director of Group Sales, • Print Semester Program: Paid subscribers who read all CME articles during each Emergency Medicine Practice 6-month testing period, complete the post-test and at 678-366-7933 or the CME Evaluation Form distributed with the June and December issues, and return it according to the published instructions are eligible for up to 4 hours of CME credit for each issue. You must complete both the post-test and CME Evaluation Form to [email protected] for receive credit. Results will be kept confidential. CME certificates will be delivered to each participant scoring higher than 70%. more information, or visit • Online Single-Issue Program: Current, paid subscribers who read this Emergency Medicine Practice CME article and complete the online post-test and CME www.ebmedicine.net/groups. Evaluation Form at www.ebmedicine.net are eligible for up to 4 hours of Category 1 credit toward the AMA Physician’s Recognition Award (PRA). You must complete both the post-test and CME Evaluation Form to receive credit. Results will be kept confidential. CME certificates may be printed directly from the website. Hardware/Software Requirements: You will need a Macintosh or PC to access the Coming In Future Issues online archived articles and CME testing. Adobe Reader is required to view the PDFs of the archived articles. Adobe Reader is available as a free download at www.adobe.com. Hypertension Additional Policies: For additional policies, including our statement of conflict of interest, source of funding, statement of informed consent, and statement of human and animal Postpartum Complications rights, visit http://www.ebmedicine.net/policies.

CEO: Robert Williford President and Publisher: Stephanie Ivy Associate Editor: Dorothy Whisenhunt Associate Editor and CME Director: Jennifer Pai Director of Member Services: Liz Alvarez Marketing & Customer Service Coordinator: Robin Williford Direct all questions to: Subscription Information: EB Medicine 48 AMA PRA Category 1 CreditsTM, 48 ACEP Category 1 credits, 1-800-249-5770 or 1-678-366-7933 48 AAFP Prescribed credits, and 48 AOA Category 2B Fax: 1-770-500-1316 CME credits, and full online access to searchable archives 5550 Triangle Parkway, Suite 150 and additional CME: $329 Norcross, GA 30092 E-mail: [email protected] Individual issues, including 4 CME credits: $30 Website: www.ebmedicine.net (Call 1-800-249-5770 or go to To write a letter to the editor, please email: [email protected] http://www.ebmedicine.net/EMP issues to order)

Emergency Medicine Practice (ISSN Print: 1524-1971, ISSN Online: 1559-3908) is published monthly (12 times per year) by EB Practice, LLC, d.b.a. EB Medicine (5550 Triangle Parkway, Suite 150, Norcross, GA 30092). Opinions expressed are not necessarily those of this publication. Mention of products or services does not constitute endorsement. This publication is intended as a general guide and is intended to supplement, rather than substitute, professional judgment. It covers a highly technical and complex subject and should not be used for making specific medical decisions. The materials contained herein are not intended to establish policy, procedure, or standard of care. Emergency Medicine Practice is a trademark of EB Practice, LLC. Copyright © 2010 EB Practice, LLC, d.b.a. EB Medicine. All rights reserved. No part of this publication may be reproduced in any format without written consent of EB Practice, LLC. This publication is intended for the use of the individual subscriber only and may not be copied in whole or part or redistributed in any way without the publisher’s prior written permission — including reproduction for educational purposes or for internal distribution within a hospital, library, group practice, or other entity.

May 2010 • EBMedicine.net 24 Emergency Medicine Practice © 2010 EVIDENCE-BASED practice RECOMMENDATIONS An Evidence-Based Approach To Traumatic Ocular Emergencies Alteveer J, Lahmann B. May 2010, Volume 12; Number 5 This issue of Emergency Medicine Practice provides an up-to-date review of the appropriate evaluation of adults with blunt and pene- trating ocular and periorbital injuries and offers evidence-based management recommendations. For a more detailed discussion of this topic, including figures and tables, clinical pathways, and other considerations not noted here, please see the complete issue on the EB Medicine website at www.ebmedicine.net/topics.

Key Points Comments Always maintain a high index of suspicion for open-globe Globe rupture is a serious injury and a major cause of monocu- injury. The patient will often, but not always, complain of pain lar blindness and must be treated promptly. In gathering data, and decreased visual acuity. remember that conjunctival lacerations may indicate underly- ing injury to the sclera and that hemorrhagic chemosis may indicate orbital fracture or open-globe injury.14

Always measure the intraocular pressure (IOP) – except in the Low IOP may be seen in occult globe rupture or laceration, suspected or obvious open-globe injury. injury to the ciliary body, or retinal detachment. Normal or even high IOP does not rule out open-globe injury or rupture. Elevated IOP may occur immediately after contusion to the globe, in the presence of cells in the anterior chamber, mechan- ical angle closure, and with anterior dislocation of the lens.14 Patients with previous surgery or injury have a higher inci- In a retrospective analysis of 100 consecutive open-globe dence of open-globe injury. injuries at a single institution, Man and Steel compared CART and OTS predictions with actual visual outcomes and calculated the sensitivity and specificity of each model. The variables most closely predictive of poor visual outcome were RAPD, poor ini- tial vision, lid laceration, posterior wound, and globe rupture. 25

Think of posterior segment injuries when presented with ante- Motor vehicle occupants 66 years of age and older were found rior segment injuries that resulted from a high-velocity insult to be 2 to 3 times more likely to incur an eye injury after airbag such as an airbag. deployment. In addition, patients who have undergone refrac- tive surgeries such as RK, PRK, and LASIK are also at an increased risk of posterior segment injury.50

Penetrating injuries in the rural setting have a higher rate of The incidence of endophthalmitis after an open-globe injury has endophthalmitis. been reported to be between 3.3% and 16.5%. Traditional risk factors included delayed primary repair/wound closure, rural set- ting, presence of retained intraocular foreign body (IOFB), and disruption of the lens.61,62 A recent large consecutive case series treated at 1 institution over 7 years showed a rate of endophthal- mitis of 0.4% without IOFB and 3.2% with IOFB.63 Dog bites to the eye have a high incidence of canalicular A high index of suspicion needs to be maintained for any lac- injury. eration/avulsion in the area of the medial canthus. Evaluation of the canalicular structures is performed by the ophthalmolo- gist and requires an awake and cooperative patient. The repair may be carried out 24 to 48 hours after the initial injury. 68

See reverse side for reference citations. 5550 Triangle Parkway, Suite 150 • Norcross, GA 30092 • 1-800-249-5770 or 678-366-7933 Fax: 1-770-500-1316 • [email protected] • www.ebmedicine.net REFERENCES

These 14. Harlan JB, Pieramici DJ. Evaluation of patients with ocular trauma. Ophthalmol Clin N Am. 2002;15:153-161. (Review article) references are excerpted from 25. Man CYW, Steel D. Visual outcome after open-globe injury: a comparison of two prognostic models – ocular trauma score and the classification and regression tree.Eye . 2010;24:84-89. Published online Feb 20, 2009. (Retrospective cohort study) the original manuscript. 50. Kenney KS, Fanciullo LM, Automobile air bags: friend or foe? A case of air bag-associated ocular trauma and a related literature review. Optometry. 2005;76:382-386. (Case report) For additional 61. Chaudhry IA, Shamsi FA, Al-Harthi E, et al. Incidence and visual outcome of endophthalmitis associated with references and intraocular foreign bodies. Arch Clin Exp Ophthalmol. 2008;246:181-186. (Retrospective case review) information on 62. Lieb DF, Scott IU, Flynn HW Jr, Miller D, Feuer WJ. Open-globe injuries with positive intraocular cultures: fac- this topic, see tors influencing final visual acuity outcomes.Ophthalmology . 2003;110(8):1560-1566. (Retrospective case series)

the full text 63. Andreoli CM, Andreoli MT, Klock CE, et al. Low rate of endophthalmitis in a large case series of open-globe inju- article at ries. Ophthalmology. 2009;147:601-608. (Retrospective case series) ebmedicine.net. 68. Long J, Tann T. Adnexal trauma. Ophthalmol Clin N Am. 2002;15:179-184. (Review article)

CLINICAL RECOMMENDATIONS Designed for Use The Evidence-Based Practice Recommendations On The Reverse Side For: use in every- • Discussions with colleagues • Preparing for the boards day practice • Developing hospital guidelines • Storing in your hospital’s library • Posting on your bulletin board • Teaching residents and medical students

Emergency Medicine Practice subscribers: Are you taking advantage of all your subscription benefits? Visit your free online account at www.ebmedicine.net to search archives, browse clinical resources, take free CME tests, and more.

Not a subscriber to Emergency Medicine Practice? As a subscriber, you’ll benefit from evidence-based, clinically rele- vant, eminently usable diagnostic and treatment recommendations for everyday practice. Plus, you’ll receive up to 192 AMA PRA Category 1 CreditsTM; 192 ACEP Category 1 credits; 192 AAFP Prescribed credits; and 192 AOA category 2B CME credits and full online access to our one-of-a-kind online database. Visit www.ebmedicine.net/subscribe or call 1-800-249-5770 to learn more today.

Questions, comments, suggestions? To write a letter to the editor, email: [email protected] For all other questions, contact EB Medicine: Phone: 1-800-249-5770 or 678-366-7933 Fax: 1-770-500-1316 Address: 5550 Triangle Parkway, Suite 150 / Norcross, GA 30092

Emergency Medicine Practice (ISSN Print: 1524-1971, ISSN Online: 1559-3908) is published monthly (12 times per year) by EB Practice, LLC, d.b.a. EB Medicine. 5550 Triangle Parkway, Suite 150, Norcross, GA 30092. Opinions expressed are not necessarily those of this publication. Mention of products or services does not constitute endorsement. This publication is intended as a general guide and is intended to supplement, rather than substitute, professional judgment. It covers a highly technical and complex subject and should not be used for making specific medical decisions. The materials contained herein are not intended to establish policy, procedure, or standard of care. Emergency Medicine Practice is a trademark of EB Practice, LLC, d.b.a. EB Medicine. Copyright © 2010 EB Practice, LLC. All rights reserved.